Sunteți pe pagina 1din 67

1.

a 25 year old male with no known medical history suddenly collapsed while playing vigorous game
of ultimate Frisbee. His friends
immediately started CPR and
called 118. The paramedics
arrived within 5 minutes and
found him in VF. He was
defibrillated successfully with
one shock with return of
spontaneous circulation. He
was transported to the
hospital for subsequent care.
ECG was obtained upon arrival
to the hospital:
What is the most likely diagnosis?
a. Long QT syndrome
b. Brugada syndrome
c. Short QT syndrome
d. Timothy syndrome
e. Cathecolaminergic polymorphic VT
2. a 25 year old male with no
known medical history
suddenly collapsed while
playing vigorous game of
ultimate Frisbee. His friends
immediately started CPR
and called 118. The
paramedics arrived within 5
minutes and found him in
VF. He was defibrillated
successfully with one shock
with return of spontaneous
circulation. He was transported to the hospital for subsequent care.
This patient makes a complete neurologic recovery.
An echocardiogram is within normal limits. What is the next appropriate step in management?
a. EP testing with administration of a class 1 antiarrhythmic (flecainide and procainamide) to
determine risk of sudden death
b. Implant a dual-chamber pacemaker
c. Exercise testing to assess if this QT shortens appropriately
d. Start a beta blocker and restrict him from participation in competitive sports
e. Implant an ICD
3. A 29-year-old woman with known
insulin-dependent diabetes mellitus
was found unconscious 1 hour after
an office party. Initial assessment
by the emergency medical service
team showed a BP of 90/60 mmHg.
Her pulse was 120, and her blood
sugar was 870 mg/dL. She was
given SC insulin and rushed to the emergency department. You are called to see her because of her
abnormal ECG. She is noted to be semiconscious. The emergency physician has already started her
on IV insulin drip and hydration.
What is your recommendation at this juncture?
a. Her ECG predicts high-degree atrioventricular block; a standby external pacemaker should
be available.
b. She has ECG evidence of hyperkalemia, and she needs IV calcium and, possibly dialysis
c. She is having an acute MI, and immediate restoration of coronary flow is essential
d. Continue the current management; the ECG will improve with the resolution of
ketoacidosis.
4. A 40-year-old woman is referred to your office for evaluation of a murmur heard during a routine
physical examination. She is asymptomatic. She used to jog 2 to 3 miles day without problems but
over the past few years has stopped exercising. She had frequent febrile illnesses as a child, but her
past medical history is otherwise
unremarkable. Physical examination
-. BP 120/70 mmHg, pulse 73 bpm
-. She is in no acute distress.
-. Jugular venous pulse (JVP) is not elevated.
-. Chest is clear
-. Chardiac-PMI not displaced
- . R e g u l a r r a t e
is heard at rest and is heard best
between the apex and left sternal
border, 0.10 seconds after S2. This is
followed by a low-pitch decrescendo
murmur with pre-systolic accentuation.
-. Abdomen-no organomelgaly
-. Extremities-no Edema. Normal distal pulses. Good capillary refill.
An echocardiogram is performed; proximal flow convergence radius (PFCR) using color 3D across the
mitral valve indicates an orifice area of 1.2 cm 2. Resting PA pressures are 35 mmHg. Splittability
score is 5. LV size and function are normal.
a. Follow-up in 2 years
b. Stress echocardiogram, to assess for mitral pressures post stress
c. Immediate referral for percutaneuous valvuloplasty
d. Immediate referral for surgery
5. A 42-year-old male nurse in a rural emergency room develops crushing chest pain 30 minutes after
he smoked a cigarette during a break in
his shift. An ECG is performed.
What is the most appropriate
intervention?
a. Initiate fibrinolytic therapy
with reteplase with
adjunctive treatment with
aspirin and intravenous
heparin
b. Treat with intravenous
metoprolol 5 mg three
times
c. Load with clopidogrel 600 mg and initiate heparin for a diagnosis of NSTEMI ACS
d. Treat with intravenous verapamil for vasospasm
e. Load with prasugrel 60 mg once daily and transfer for primary PCI to a laboratory 4
hours away.
6. A 42-year-old mother is visiting her child who is undergoing chemotherapy in the Children’s
Hospital. She suddenly develops chest pain and is noted to be nauseous and diaphoretic. An ECG is
performed (Fig. 3-1). On angiography, the
coronaries are free of significant epicardial
stenosis. A venticulogram shows apical
ballooning with an LVEF of 25%.
The patient is admitted to the CCU.

Which of the following statements will


most likely define her clinical course?
a. Indefinite anticoagulation
with warfarin will be indicated.
b. Her prognosis will likely be excellent with full recovery of LV function
c. She is at higher risk for a malignancy in the ensuing year
d. She is at risk for recurrent atherosclerotic ischemic events
e. She will likely require an ICD as LV function is unlikely to recover
7. A 57-year-old woman presents for the evaluation of insidious exertional dyspnea.
Shown is a clip from her TEE:

Which of the following is


most likely?
a. Hypernephroma
b. Myxoma
c. Papillary fibroelastoma
d. Thrombus
e. Pericardial cyst
8. A 59-year-old man with a history of coronary artery disease (CAD) and remote coronary bypass
surgery presents with progressive dyspnea and vague chest pain, he had a stress echocardiogram for
these symptoms that demonstrated
normal LV function with no stress-
induced wall motion abnormalities.
However he returned to the
emergency department a few days
later with recurrent symptoms. This
time the house officer examining
the patient notes 3+ pedal edema.
The patient is admitted and started
on diuretics. His blood tests are as
follows
WBC count = 11.000
HB = 14.2
PLT = 172.000
Albumin = 4.6
Urea = 11
Creatinine = 0.9
Owing to the reccurent symptoms, his cardiologist decides to refer him for a right and left heart
catheterization. The coronary grafts are all patent. The tracing from the study are shown.
What is the most logical explanation for this patient’s symptoms?
a. Small-vessel CAD
b. Cardiac amyloid
c. Constrictive pericardial disease
d. Cardiac tamponade
e. Diastolic dysfunction related to his chronic CAD
9. A 59-year-old woman with a history
of chronic renal insufficiency
presents to the emergency
department with anterior left-sided
chest pain. She reports that the
chest pain started after her last
dialysis 7 days ago. She appears
lethargic and in mild respiratory
distress. The physical examination
demonstrates a BP of 160/90 mmHg
and an HR of 100 bpm. On cardiac auscultation, a loud friction rub is heard. An ECG is obtained.
What is the most important next step in this case?
a. Perform emergency dialysis
b. Obtain an echocardiogram
c. Prepare for pericardiocentesis
d. Admit the patient to the cardiac care unit to rule out MI
10. A 64-year-old man presents with ST elevation and is taken for primary angioplasty. Diagnostic
angiogram in RAO 9 CRA 39 projection is shown
Identify the infarct-related artery?
a. Obtuse marginal
b. Diagonal
c. Left circumflex
d. LAD
e. RCA

11. A 65-year-old woman presents to your office for follow-up


of a murmur she was told
about several years prior.
She denies any symptoms,
but is not very active. Her
past medical history is
significant for hypertension
and diabetes, both of which
have been well controlled.
On examination, she is in no
acute distress. BP is 125/75
mmHg, with a resting heart
rate of 70 bpm. Lungs are
clear. Cardiac examination
reveals a displaced PMI. S1 is
soft. S2 reveals an increased
P2 component. There is a
right ventricular (RV) lift. An
S3 is present. There is a
grade III/VI holosystolic murmur heard at the apex radiating to the base. She has no peripheral
edema. Chest X-Ray demonstrated cardiomegaly with prominence of the central pulmonary
vasculature.
An echocardiogram is performed on this patient. Left ventricular (LV) systolic dimension is 4,7 cm.
ejection fraction is 45%. There is posterior leaflet prolapse. There is a very eccentric jet of MR, which
is read out as 2+.
Which of the following is most likely?
a. Her LV function is better than it appears on the echocardiogram
b. She likely has more severe MR than is evident on the echocardiogram
c. MR is unlikely to account for her presentation
d. TEE is unlikely to be helpful here
12. A 67-year-old woman is admitted to the emergency room because of sudden onset of chest pain and
rapid pulse. She has no history of similar occurrence. Physical examination reveals a pale diaphoretic
woman in moderate respiratory distress. Her blood pressure is palpable at 75 mmHg systolic. Lungs
show bibasilar crackles. There is no jugular venous distention and heart sounds are distant with a
variable S1. A 12 lead electrocardiogram is shown.
What is the appropriate immediate therapy?
a. Direct current cardioversion
b. Rapid infusion of 250 ml
of normal saline
c. Amiodarone, 300 mg
intravenously over 300
min
d. Digoxin, 0,50 mg
intravenously
e. Diltiazem, 20 mg
intravenously, followed
by 10 mg/hr infusion

13. A 75-year-old woman presented with shortness of breath and generalized malaise of 1-week
duration. Her heart rate was 90 per
minute and BP was 90/50 mmHg.
Examination revealed rales in bilateral
lung bases, an elevated jugular venous
pulse, and a loud systolic murmur in the
left parasternal area. An echo was
performed.
What is the likely infarct-related artery?
a. Proximal LAD
b. Proximal nondominant RCA
c. Mid-RCA in dominant RCA
d. Obtuse marginal in a
dominant circumflex
e. Distal LAD

14. According to the ASE guideline, measuring the Aorta dimension should be:
a. From A to D
b. From B to D
c. Either A or D
d. From A to C
e. From B to C

15. All of following statements regarding


the figure above are correct EXCEPT:
a. Position of the image intensifier is left to the midline and toward the feet of the patient
b. An important view for the distal left main stem and the proximal segments of the LAD
and circumflex
c. Spider view
d. A best view o assess the distal segment of LAD and circumflex, the diagonal and septal
branch of LAD
16. The channelopathy underlying the clinical presentation in patient history of cardiac arrest is:
a. Loss of function in the sodium channel
b. Gain of
function
in the
calcium
channel
c. Gain of
function
in the
sodium
channel
d. Gain of
function
in the potassium channel
e. Loss of function in the potassium channel
17. The echocardiography was performed the pressured half time of the mitral valve was measured at
440 ms. MVA for this patients:
a. 1.1 cm2
b. 1.5 cm2
c. 2.0 cm2
d. Cannot calculate with
information available
e. 0.5 cm2

18. This M-mode tracing demonstrates a patient


with:
a. Rheumatic mitral stenosis
b. HOCM with systolic
anterior motion (SAM) of
the MV leaflets
c. Severe left ventricular (LV)
dysfunction.
d. MVP
e. AS
19. What is the diagnosis from the ECG and pressure tracing below?
a. ischemic cardiomyopathy
b. pericardial
tamponade
c. Restrictive
cardiomyopathy
d. Constrictive
pericarditis

20. What is the diagnosis of the following cardiac catheterization still-frame


sides?
a. Pulmonic stenosis
b. Patent ductus arteriosus (PDA)
c. Coarctation of the aorta
d. VSD
e. Hypertrophic cardiomyopathy

21. What is the diagnosis from the pressure tracing below?


a. Severe AR
b. Severe LV systolic dysfunction
c. HCM
d. Severe AS
22. What is the diagnosis of the following cardiac catheterization still-frame slide?
a. Patent ductus arteriosus (PDA)
b. VSD
c. Pulmonic stenosis
d. Hypertrophic cardiomyopathy
e. Coarctation of the aorta

23. What is the diagnosis of the following cardiac catheterization still-


frame slide?
a. Hypertrophic cardiomyopathy
b. Coarctation of the aorta
c. Patent ductus arteriosus (PDA)
d. Pulmonic stenosis
e. VSD

24. What is the diagnosis of the schematic illustrations of the


carotid pulse?
a. Aortic stenosis
b. Hypertrophic cardiomyopathy
c. Severe CHF decompensation
d. Normal
e. Aortic regurgitation

25. With regard to the diagram (Figure 1). Which of the following points or curves will move first if a
patient develops an infiltrative process such as amyloid heart disease?
a. Curve E
b. Curve B
c. Curve D
d. Curve A
e. Curve C
26. With which of the following adult congenital heart conditions can the following ECG tracing be seen?
a. VSD
b. Coarctation of the aorta
c. Ebstein
anomaly
d. Primum
ASD
e. Congenital
corrected

transposition

1. 10 day-old baby boy is admitted to PICU with severe cyanoses. The parents and the midwives have
noted the appearance of their baby since he was born. The parents did not bring the baby to the tertiary
hospital because they refused to be referred. In the last 24 hours the baby looked very sick and fatigue
so that finally he was brought to our hospital. On physical examination the baby is cyanotic with poor
periphery perfusion. He was intubated and saturating at 45%. He got a normal SI with enhancement of
S2. Continuous murmur grade III/6 heard ICS2 LSB. No gallop is heard. On chest X-ray the lungs is
oligemic.
Which of the following is a possible diagnosis differential:
a. Edward Syndrome
b. AP window
c. A lesion of Duct dependent pulmonary circulation
d. Patent Ductus Arteriosus
e. Ventricle septal defect

2. 10 day-old baby boy Is admitted to PICU with severe cyanoses. The parents and the midwives have
noted the appearance of their baby since he was born. The parents did not bring the baby to the tertiary
hospital because they refused to be referred. In the last 24 hours the baby looked very sick and fatigue
so that finally he was brought to our hospital. On physical examination the baby is cyanotic with poor
periphery perfusion. He was intubated and saturating at 45%. He got a normal SI with enhancement of
S2. Continuous murmur grade III/6 heard ICS2 LSB. No gallop is heard. On chest X-ray the lungs is
oligemic.
Which of the following is a possible diagnosis differential:
a. Ventricle septal defect
b. A lesion of Duct dependent pulmonary circulation
c. Patent Ductus Arteriosus
d. AP window
e. Edward Syndrome

3. a 17-year-old patient who is known to have Wolff-Parkinson-White syndrome presents with a regular
narrow complex tachycardia with a cycle length of 375 milliseconds (160 bpm) that occurred with a
sudden onset. You note that there is a 1:1 atrial-to-ventricular relationship and that the RP interval is
100 milliseconds.
The best initial treatment is
a. catheter ablation
b. atropine
c. IV procainamide
d. vagal maneuver

4. a 45-year old woman presents with DOE and paroxysmal AF. She has a fixed split S2 on exam with a
2/6 SEM at the LSB. Echocardiogram shows:
-. Dilated RV and RA, normal LV
-. Secundum ASD
-. PA systolic, 70 mmHg
The following are found at catheterization:
• Saturations (%): IVC, 70; SVC, 67; RA, 82; RV, 87; PA, 88; LV, 98; aorta, 98; FA, 98
• Pressures (mmHg): PA, 70/50; FA, 120/70; PCWP, 10
• TCO, 6.0 L/min
What is the Qp/Qs?
a. Need more information
b. 1.5:1.0
c. 2.5:1.0
d. 2.0:1.0
e. 3.0:1.0

5.??????????????????????????????????????????????????????????????????????????????????????????
The patients dyspnea began 50 minutes ago. On physical examination, her pulse rate is 110/min
respiration rate is 34/min, and blood pressure Is 100/60 mm Hg. Jugular venous pressure is elevated at
10 cm H20, crackles are heard halfway up both lung fields, a parasternal lift is appreciated, and there is a
new grade 3/6 systolic murmur at the left sternal border with an S3 gallop. The electrocardiogram shows
sinus tachycardia with Q waves and T wave inversions in leads n, m, and aVF, and is unchanged from the
discharge electrocardiogram.
A pulmonary artery catheter is placed, which shows the following:
Pressure (mmHg) Oxygen Saturation (%)
Right atrium 12 (normal 2-7) 49 (normal 60-75)
Right ventricle 60/12 (normal 20-30/2-7) 78 (normal 60-75)
Pulmonary artery 60/32 (normal 20-30/10-15) 80 (normal 60-75)
Pulmonary capillary wedge 24 (normal <14) 98 (normal >93)
Which of the following is the most likely diagnosis?
a. Atrial septal defect
b. Pericardial tamponade
c. Ventricular septal defect
d. Papillary muscle rupture
e. Recurrent myocardial infarction

6. HF prognosis correlates MOST STRONGLY with which of the following?


a. Resting EF
b. Genetic predisposition
c. NYHA class
d. Diastolic dysfunction
e. Impaired renal function

7. An 80-year-old man presents to your office with complaints of chest tightness when climbing up a
flight of stairs. His past medical history is unremarkable. On physical examination, he is in no acute
distress. BP is 140/80 mmHg; pulse is 78 bpm and regular. Chest is clear. Carotid upstrokes are
diminished. The PMI is sustained, but not displaced. A fourth heart sound is present. The second heart
sound is diminished and single. A bud late-peaking systolic murmur is heard, loudest at the second
intercostal space, radiating to the neck.
The above patient is found to have an aortic valve area of 0.7 cm2 with a mean gradient of 60 mmHg.
Following catheterization, he develops massive upper gastrointestinal bleeding. Endoscopy reveals a
gastric ulcer with a bleeding vessel at its base. Cauterization is performed, which temporarily stops the
bleeding. However, the bleeding recurs and urgent partial gastrectomy is recommended. He complains
of chest pain during these bleeding episodes.
What is the best course of action?
a. Refer for percutaneous balloon valvuloplasty, followed by gastrectomy.
b. Proceed to AVR first.
c. Start nitroprusside and proceed with gastric surgery.
d. Proceed with gastric surgery directly.

8. A 62-year-old man with a history of rheumatic heart disease presents to your office with complaints of
exertional dyspnea. No constitutional complaints are present. He had undergone a mitral valve
replacement with a bileaflet tilting disk mechanical valve 11 years prior. He is normotensive with a heart
rate of 73 bpm. On examination, you note a grade II/VI holosystolic murmur at the apex. An
echocardiogram is performed, which reveals normal LV and RV function. Peak mitral gradient is 30
mmHg. Mean transmitral gradient is 7 mmHg. Pressure half-time is 80 milliseconds.
What is your next diagnostic step?
a. Invasive assessment of hemodynamics
b. Transesophageal echocardiogram
c. Fluoroscopy of the valve
d. Draw blood cultures

9. 68-year-old woman presents to your office for initial evaluation. She has had progressive dyspnea
over the past 2 years. She has long-standing hypertension and reports tobacco use (50 pack-years) she
has been treated with bosentan for idiopathic PAH (iPAH). Currently, she is assessed as having World
Health Organization (WHO) Functional class III limitations.
Evaluation revels the following:
Echocardiogram: ejection fraction 66%, grade 3 diastolic abnormality, moderate left ventricular
hypertrophy, no significant valvular disease
Right heart catheterization: right atrial pressure 8 mmHg, right ventricular pressure 45/20 mm Hg,
pulmonary artery pressure 50/24 mmHg, mean pulmonary artery pressure 33 mmHg, pulmonary
capillary wedge pressure 25 mmHg, and cardiac output 5.5 L/min
Based on your review of her evaluation findings,
Which of the following represents the appropriate diagnosis?
a. Pulmonary Vein hypertension
b. Chronic obstructive pulmonary disease (COPD)
c. Pulmonary arterial hypertension
d. Viral cardiomyopathy

10. 68-year-old woman presents to your office for initial evaluation. She has had progressive dyspnea
over the past 2 years. She has long-standing hypertension and reports tobacco use (50 pack-years) she
has been treated with bosentan for idiopathic PAH (iPAH). Currently, she is assessed as having World
Health Organization (WHO) Functional class III limitations.
Evaluation revels the following:
Echocardiogram: ejection fraction 66%, grade 3 diastolic abnormality, moderate left ventricular
hypertrophy, no significant valvular disease
Right heart catheterization: right atrial pressure 8 mmHg, right ventricular pressure 45/20 mm Hg,
pulmonary artery pressure 50/24 mmHg, mean pulmonary artery pressure 33 mmHg, pulmonary
capillary wedge pressure 25 mmHg, and cardiac output 5.5 L/min
Which of the following treatments would you recommend at this time?
a. Increase bosentan dose
b. Phosphodiesterase-5 inhibitor
c. Cardiac resynchronization therapy
d. Diuretics, salt restriction, and blood pressure control

11. 68-year-old woman presents to your office for initial evaluation. She has had progressive dyspnea
over the past 2 years. She has long-standing hypertension and reports tobacco use (50 pack-years) she
has been treated with bosentan for idiopathic PAH (iPAH). Currently, she is assessed as having World
Health Organization (WHO) Functional class III limitations.
Evaluation revels the following:
Echocardiogram: ejection fraction 66%, grade 3 diastolic abnormality, moderate left ventricular
hypertrophy, no significant valvular disease
Right heart catheterization: right atrial pressure 8 mmHg, right ventricular pressure 45/20 mm Hg,
pulmonary artery pressure 50/24 mmHg, mean pulmonary artery pressure 33 mmHg, pulmonary
capillary wedge pressure 25 mmHg, and cardiac output 5.5 L/min
Which of the following scenarios may occur upon acute vasodilator challenge (pulmonary hypertension
reactivity test) in this patient?
a. Altered mental status
b Acute life-threatening pulmonary edema in the setting of a markedly elevated pulmonary
capillary wedge pressure
c. The patient may not respond and can then be considered a candidate for chronic calcium
channel blocker therapy
d. Acute renal failure

12. Which of the following drugs can significantly increase digoxin concentrations?
a. Metoprolol
b. Simvastatin
c. Amiodarone
d. Fenofibrate

13. A 2-year-old-boy is brought into the emergency room with a complaint of fever for 6 days and the
development of a limp. On examination, he is found to have an erythematous macular exanthem over
his body as shown in image A, ocular conjunctivitis, dry and cracked lips, a red throat, and cervical
lymphadenopathy. There is a grade 2/6 vibratory systolic ejection murmur at the lower left sternal
border. A white blood cell (WBC) count and differential show predominant neutrophils with increased
platelets on smear.
Which of the following is the most likely diagnosis?
a. Infectious mononudeosus
b. Kawasaki disease
c. Scarlet Fever
d. Juvenille rheumatid arthritis
e. Rheumatic Heart Fever

14. A 15-year-old girl is being evaluated for a heart murmur. She is asymptomatic. On physical
examination, her blood pressure is 174/104 mm Hg on her right arm. Her pulses are 2+ on her upper
extremities. The femoral pulses are slightly lower in amplitude than the radial pulses. Her cardiac
examination reveals a short midsystolic murmur in the left infrascapular area.
For this patient,
which of the following is most likely to be found on additional studies?
a. Stenotic pulmonic valve on echocardiogram
b. Cardiomegaly and pulmonary engorgement on chest x-ray
c. Downward displacement of the tricuspid valve annulus toward the right ventricle apex on
echocardiogram
d. Rib notching and dilatation of the aorta on chest x-ray

15. A 20-year-old college student is evaluated for palpitations. She has noticed a rapid pounding in her
chest on several occasions. She first noticed it in middle school. Most episodes last less than 1 minute,
but a few have lasted one-half hour. During an episode, she feels lightheaded, but is not short of breath
and does not have syncope or chest pain. She is most aware of the pounding sensation in her neck.
Symptoms usually occur without warning while she is at rest. If she breathes slowly and deeply, the
episodes usually stop on their own. They have never interfered with her activities, and she continues to
run cross-country. Recently, the episodes have been longer and more frequent. She has never had the
rhythm documented. Physical examination and electrocardiogram findings are normal. She has no
family history of heart disease.
Which of the following is the most likely diagnosis?
a. Palpitations related to mitral valve prolapse
b. Paroxysmal atrial flutter
c. Benign premature atrial contractions
d. Ventricular tachycardia
e. Paroxysmal supraventricular tachycardia
16. a 20-year-old woman who has a family history of marfan syndrome comes to you for prepregnancy
counseling. Echocardiogram and CT scan of the chest show 42-mm dilation of the aortic sinuses. The
patient is asymptomatic and does not have mitral valve prolapse.
Which of the following is the most appropriate recommendation?
a. Avoid pregnancy at this time because of the size of the aorta, and initiate treatment with
a ?-blocker.
b. Initiate treatment with a ?-blocker, and proceed with pregnancy.
c. Recommend urgent aortic surgical intervention now.
d. Avoid pregnancy because of the autosomal dominant nature of Marfan syndrome.
e. Proceed with pregnancy, preferably sooner rather than later because of the size of the aorta.

17. A 20-year-old woman who is in the first trimester of her first pregnancy is referred to you because of
progressive dyspnea. An echocardiogram shows findings consistent with severe pulmonary
hypertension. The right ventricular systolic pressure is 90 mm Hg, and systemic blood pressure is 110
mm Hg. Further studies show no evidence of pulmonary embolism. Transesophageal echocardiogram
excludes an intracardiac shunt.
Which of the following is associated with the best risk of maternal mortality?
a. Termination of pregnancy
b. Initiation of ACE inhibitor therapy
c. Initiation of anticoagulant therapy
d. Initiation of prostacyclin therapy
e. Initiation of bosentan therapy 

18. a 21-year-old intravenous drug abuser came to the hospital with fever and malaise of 3 weeks’
duration. His chest X-ray showed right –sided pneumonia and bilateral plural effusions. The initial
examination was remarkable for inspiratory wheezes and dullness at the right base. There were
prominent c-v waves in his jugular venous pulse and a hyperdynamic precordium was noted. A grade 3/6
systolic murmur was audible most prominently along the left sternal border. Five of six blood cultures
were positive for Pseudomonas aeruginosa and he was started on tobramycin and piperacillin. However,
after 3 weeks he had persistent fever to 43°C and 1+ pedal edema, and blood cultures were again
positive for Pseudomonas.
Appropriate therapy at this time would be:
a. antibiotics changed to tobramycin and ceftazidime and given for an additional 4 weeks
b. tricuspid valvulectomy with replacement by a prosthetic valve and antibiotic treatment for 4
weeks
c. tricuspid valvulectomy with replacement by a Hancock porcine prosthesis and antibiotics for 4
weeks
d. tricuspid valvulectomy with replacement by a St. Jude mechanical prosthesis and continued
antibiotics for 4 weeks
e. an additional 4 weeks of treatment with higher doses of tobramycin and piperacillin

19. A 23-year-old woman with Down syndrome (trisomy 21) presents to establish care. In general, she
appears comfortable. There is no evidence of cyanosis. Her cardiac examination reveals a soft systolic
murmur with a midsystolic click. The clinical diagnosis of mitral valve prolapse is made. The patient's
mother wonders if there are any other cardiac issues.
Which congenital heart anomaly is this patient most likely to have?
a. PDA
b. Tetralogy of Fallot
c. Ostium primum defect
d. Transposition of great vessels
e. Pulmonary stenosis

20. A 25 years old pregnant woman (G1P0A0, gestational age 32 weeks) was admitted to the hospital
with main complaint shortness of breath and orthopnea. She never complained this symptoms before.
Patient also complain hemoptysis.
On physical examination, Blood pressure 100/60 mmHg, Heart rate was 132 x/minute irregularly
irregular, Respiratory rate 28 breaths/minute. Patient had distended jugular vein and left parasternal
heave. Cardiac auscultation revealed low pitched rumbling mid diastolic murmur.
Chest X ray revealed congested upper lobe vein and Kerley B lines.
Which of the following is the most appropriate treatment to control the heart rate?
a. Digoxin
b. Flecainide
c. Amiodarone
d. Methyl Dopa
e. Verapamil

21. A 25 years old pregnant woman (G1P0A0, gestational age 32 weeks) was admitted to the hospital
with main complaint shortness of breath and orthopnea. She never complained this symptoms before.
Patient also complain hemoptysis.
On physical examination, Blood pressure 100/60 mmHg, Heart rate was 132 x/minute irregularly
irregular, Respiratory rate 28 breaths/minute. Patient had distended jugular vein and left parasternal
heave. Cardiac auscultation revealed low pitched rumbling mid diastolic murmur.
Chest X ray revealed congested upper lobe vein and Kerley B lines.
Which one of the following is the most likely anatomic diagnosis for this patient?
a. Aortic Stenosis
b. Pulmonary Stenosis
c. Tricuspid stenosis
d. Mitral stenosis
e. Aortic Regurgitation

22. A 25 years old pregnant woman (G1P0A0, gestational age 32 weeks) was admitted to the hospital
with main complaint shortness of breath and orthopnea. She never complained this symptoms before.
Patient also complain hemoptysis.
On physical examination, Blood pressure 100/60 mmHg, Heart rate was 132 x/minute irregularly
irregular, Respiratory rate 28 breaths/minute. Patient had distended jugular vein and left parasternal
heave. Cardiac auscultation revealed low pitched rumbling mid diastolic murmur.
Chest X ray revealed congested upper lobe vein and Kerley B lines.
Which of the following is the most appropriate anti thrombotic drugs for this patients?
a. Dabigatran
b. low molecular weight heparin
c. Unfractionated heparin
d. warfarin
e. fondaparinux
23. A 25 years old pregnant woman (G1P0A0, gestational age 32 weeks) was admitted to the hospital
with main complaint shortness of breath and orthopnea. She never complained this symptoms before.
Patient also complain hemoptysis.
On physical examination, Blood pressure 100/60 mmHg, Heart rate was 132 x/minute irregularly
irregular, Respiratory rate 28 breaths/minute. Patient had distended jugular vein and left parasternal
heave. Cardiac auscultation revealed tow pitched rumbling mid diastolic murmur.
Chest X ray revealed congested upper lobe vein and Kerley B lines.
Below is predictive factors of successful intervention for this patient, EXCEPT:
a. Absent of congestive heart failure
b. Mild mitral regurgitation
c. Minimal subvalvular disease
d. Mild calcification
e. Good mobility of the valve

24. A 25 years old pregnant woman (G1P0A0, gestational age 32 weeks) was admitted to the hospital
with main complaint shortness of breath and orthopnea. She never complained this symptoms before.
Patient also complain hemoptysis.
On physical examination, Blood pressure 100/60 mmHg, Heart rate was 132 x/minute irregularly
irregular, Respiratory rate 28 breaths/minute. Patient had distended jugular vein and left parasternal
heave. Cardiac auscultation revealed low pitched rumbling mid diastolic murmur.
Chest X ray revealed congested upper lobe vein and Kerley B lines.
Which of the following is the most appropriate treatment to control the heart rate?
a. Digoxin
b. Amiodarone
c. Verapamil
d. Methyl Dopa
e. Flecainide

25. A 25 years old pregnant woman (G1P0A0, gestational age 32 weeks) was admitted to the hospital
with main complaint shortness of breath and orthopnea. She never complained this symptoms before.
Patient also complain hemoptysis.
On physical examination, Blood pressure 100/60 mmHg, Heart rate was 132 x/minute irregularly
irregular, Respiratory rate 28 breaths/minute. Patient had distended jugular vein and left parasternal
heave. Cardiac auscultation revealed low pitched rumbling mid diastolic murmur.
Chest X ray revealed congested upper lobe vein and Kerley B lines.
Which one of the following is the indication for surgery or repair in this patient?
a. Hemoptysis
b. Pulmonary artery systolic pressure <60 mmHg during exercise
c. Thromboembolic event
d. Pulmonary artery systolic pressure <50 mHg at rest
e. Valve area >1,5 cm2

26. A 25 years old pregnant woman (G1P0A0, gestational age 32 weeks) was admitted to the hospital
with main complaint shortness of breath and orthopnea. She never complained this symptoms before.
Patient also complain hemoptysis.
On physical examination, Blood pressure 100/60 mmHg, Heart rate was 132 x/minute irregularly
irregular, Respiratory rate 28 breaths/minute. Patient had distended jugular vein and left parasternal
heave. Cardiac auscultation revealed low pitched rumbling mid diastolic murmur.
Chest X ray revealed congested upper lobe vein and Kerley B lines.
Below is predictive factors of successful intervention for this patient, EXCEPT:
a. Good mobility of the valve
b. Minimal subvalvular disease
c. Mild calcification
d. Absent of congestive heart failure
e. Mild mitral regurgitation

27. A 25 year old woman presents with exertional dyspnea and orthopnea in the 30th wk of her first
pregnancy. She has a history of rheumatic fever in childhood and has not had a recent cardiac
evaluation. She is currently on no medications. Physical examination reveals a pulse of 100 bpm with a
regular rhythm. The BP is 110/76mmHg. There is mild JVD. A and V waves are visible. The lungs are clear.
Cardiac examination reveals a palpable first heart sound and a parasternal lift.
The second heart sound is somewhat increased. There is an opening snap followed by a grade 2/6
diastolic rumble noted at the apex and LSB. The ECG demonstrates sinus rhythm with LA abnormality. A
TTE is performed and this demonstrates MS.
Which of the following is most appropriate at this time?
a. Open mitral commissurotomy
b. PMBV
c. MVR
d. Institution of HR control, diuresis, and warfarin
e. Echocardiography hemodynamic study

28. 25 year old woman presents with exertional dyspnea and orthopnea in the 30th wk of her first
pregnancy. She has a history of rheumatic fever in childhood and has not had a recent cardiac
evaluation. She is currently on no medications. Physical examination reveals a pulse of 100 bpm with a
regular rhythm. The BP is 110/76mmHg. There is mild JVD. A and V waves are visible. The lungs are clear.
Cardiac examination reveals a palpable first heart sound and a parasternal lift. The second heart sound is
somewhat increased. There is an opening snap followed by a grade 2/6 diastolic rumble noted at the
apex and LSB. The ECG demonstrates sinus rhythm with LA abnormality. A TTE is performed and this
demonstrates MS.
The patient is started on medical therapy. She returns with persistent symptoms of dyspnea and
orthopnea after 1 wk of therapy. Physical examination demonstrates a HR at 65 bpm. The cardiac
examination findings are similar to those previously noted. A limited TTE is repeated. This demonstrates
similar mitral valve morphology. The resting mean gradient across the mitral valve is 12mmHg. The
calculated valve area is 1.0 cm2. The calculated RVSP is 60mmHg.
Which of the following is the most appropriate at this time?
a. Change medical therapy
b. MVR
c. Open mitral commissurotomy
d. PMBV
e. Urgent cesarean delivery

29. A 25 year old woman presents with exertional dyspnea and orthopnea in the 30th wk of her first
pregnancy. She has a history of rheumatic fever in childhood and has not had a recent cardiac
evaluation. She is currently on no medications. Physical examination reveals a pulse of 100 bpm with a
regular rhythm. The BP is 110/76mmHg. There is mild JVD. A and V waves are visible. The lungs are clear.
Cardiac examination reveals a palpable first heart sound and a parasternal lift.
The second heart sound is somewhat increased. There is an opening snap followed by a grade 2/6
diastolic rumble noted at the apex and LSB. The ECG demonstrates sinus rhythm with LA abnormality. A
TTE is performed and this demonstrates MS.
Which of the following is most appropriate at this time?
a. Institution of HR control, diuresis, and warfarin
b. MVR
b. Echocardiography hemodynamic study
d. PMBV
e. Open mitral commissurotomy 

30. A 25 year old woman who is 30 weeks pregnant is referred to you because of a murmur that was
noted during her current pregnancy as well as intermittently in the past. The patient is asymptomatic.
Physical examination shows slight elevation of the jugular venous pressure, with an A wave. A
parasternal lift is also noted. SI is normal, and S2 is somewhat prominent, fixed, and split. A grade 2 mid-
peaking ejection systolic murmur is noted at the left sternal border.
true statemement about this patient condition include all of the following EXCEPT:
a. in children this condition typically experience easy fatigability and exertional dyspnea
b. the most common presenting symptom are exercise intolerance and palpitation
c. the sinus venosus type is almost always accompanied by anomalous pulmonary venous
connections
d. atrial arrhythmia are uncommon in children with this condition
e. a patent foramen ovale can be found in approximately 25% of healthy adults 

31. A 25 year old woman who is 30 weeks pregnant is referred to you because of a murmur that was
noted during her current pregnancy as well as intermittently in the past. The patient is asymptomatic.
Physical examination shows slight elevation of the jugular venous pressure, with an A wave. A
parasternal lift is also noted. SI is normal, and S2 is somewhat prominent, fixed, and split. A grade 2 mid-
peaking ejection systolic murmur is noted at the left sternal border.
Which of the following is the most likely diagnosis?
a. Physiologic murmur related to pregnancy
b. Pulmonary valve stenosis
c. Mitral valve stenosis
d. Atrial septal defect with associated volume overload
e. Aortic valve regurgitation

32. A 28-year-old 20-week pregnant woman is referred to your clinic after being diagnosed with mitral
valve prolapse and severe MR on an echocardiogram ordered by her obstetrician. She reports no
symptoms prior to pregnancy but since being told her diagnosis is extremely worried and has noticed
some shortness of breath on exertion (New York Heart Association [NYHA] class II). She is clinically
euvolemic.
What do you recommend?
a. No therapy at present but follow carefully with serial clinical and echo evaluation
b. Antibiotics at the time of delivery
c. Commence afterload reduction with diuretics and hydralazine
d. Refer to an experienced surgeon for consideration for mitral valve repair as there is a high
likelihood of successful durable repair
e. Commence afterload reduction with an ACEI given her new onset symptoms
33. A 28-year-old man Is referred to your office for a second opinion regarding his hypertension. On
physical examination, he is in no acute distress. BP is 160/90 mmHg, symmetric in both arms. Pulse rate
is 75 bpm. Cardiac examination reveals a nondisplaced PMI. SI is normal. It is followed by a high-pitched
sound widely transmitted throughout the precordium. A short n/VI systolic ejection murmur is heard. S2
is normal.
What is the most important diagnostic test to perform next?
a. Check lower extremity BP.
b. Check plasma catecholamines.
c. Check serum potassium level.
d. Check plasma cortisol levels. 

34. A 28-year-old woman has palpitations that she describes as a "heavy beat" associated with a pause.
They have occurred sporadically over the past several years, but have become more frequent in the last
few months. She has no associated lightheadedness or syncope. Heart rate is 72/min and blood pressure
is 108/68 mm Hg. Physical examination shows a mid-systolic non-ejection click. A surface 12-lead
electrocardiogram shows sinus rhythm with a normal axis and intervals and no evidence of ventricular
preexcitation. Transthoracic echocardiography shows posterior leaflet mitral valve prolapse with mild
late-systolic mitral regurgitation. Left atrial size and left ventricular size and systolic function are normal.
A 24-hour Holter monitor shows a total of 728 isolated, unifocal premature ventricular complexes
without couplets or ventricular tachycardia.
Which of the following is the most appropriate next step in the management of this patient?
a. Order another echocardiogram in 6 months
b. Obtain a follow-up history and physical examination in 1 to 2 years
c. Order transesophageal echocardiography
d. Initiate treatment with propafenone
e. Initiate treatment with lisinopril, 10 mg/d 

35. A 28-year-old woman is referred for evaluation of persistent dyspnea related to mitral stenosis. The
patient is 30 weeks' pregnant, and her dyspnea persists despite treatment with metoprolol, furosemide,
and digoxin. Her heart rate is 70/min. An echocardiogram shows severe mitral valve stenosis, with a
mean gradient of 14 mm Hg and a mitral valve area of 1 cm 2. Trivial mitral valve regurgitation is noted.
The estimated right ventricular systolic pressure is 50 mm Hg. She has crackles in both lung bases and
bilateral lower extremity edema.
Which of the following interventions do you recommend?
a. Diagnostic cardiac catheterization
b. Surgical mitral valvotomy
c. transesophageal echocardiography followed by percutaneous mitral balloon valvuloplasty
d. Fetal echocardiogram
e. Urgent delivery of the fetus followed by reassessment of maternal cardiac status

36. A 28-year-old woman with aortic and mitral mechanical valve prostheses that were placed for
congenital heart disease comes to your office for a prepregnancy consultation. She is active and
asymptomatic. Physical examination shows normal prosthetic, aortic, and mitral valve prosthesis
auscultation. Results of laboratory studies are normal, except for a therapeutic international normalized
ratio on warfarin anticoagulation.
Which of the following recommendations is appropriate for the management of this patient during
pregnancy?
a. Continue warfarin administration throughout pregnancy and start heparin 5,000 U
subcutaneously three times daily, plus aspirin
b. Discontinue warfarin anticoagulation and initiate treatment with clopidogrel and aspirin
during the first trimester and resume treatment with warfarin for the rest of the pregnancy
until shortly before delivery
c. Discontinue warfarin administration and initiate treatment with enoxaparin, 30 mg
subcutaneously twice daily for the rest of the pregnancy
d. Discontinue warfarin and initiate dose-adjusted unfractionated heparin subcutaneously
during the first trimester and resume treatment with warfarin for the rest of the pregnancy
until shortly before delivery
e. Discontinue warfarin anticoagulation and initiate treatment with aspirin and dipyridamole for
the first trimester

37. A 29-year-old papua woman is evaluated because of severe substernal chest pain. She has not had
previous medical care, but has no known medical problems and takes no medication. Over the past 2
months, she has noted progressive fatigue, dyspnea, and palpitations. Two hours ago, she noted the
onset of severe 9/10 substernal chest pain radiating to the jaw with moderate shortness of breath. Her
blood pressure is 110/90 mm Hg and her heart rate is 110/min and irregular.
On examination, she has jugular venous distension. She has crackles one third of the way up both lung
fields. There is a parasternal heave. She has a rapid, irregularly irregular rhythm, accentuated P2, and a
n/VI early diastolic rumble at the apex. Chest radiograph shows right ventricular and left atrial
enlargement with moderate pulmonary edema.
Her electrocardiogram shows atrial fibrillation with rapid ventricular response, right bundle branch
block, right axis deviation, and 3-mm ST-segment elevation in leads V2-5.
Which of the following is the most likely cause of this patient's chest pain?
a. Coronary vasospasm.
b. Coronary arteritis.
c. Coronary thromboembolism from a left atrial thrombus.
d. Coronary thromboembolism from a left ventricular thrombus.
e. Coronary atherosclerosis with plaque rupture.

38. A 30 year-old bank executive is referred to outpatients for assessment by GP. He is known to be
hypertensive. On routine follow-up he has been found to have a systolic and diastolic murmur. He is
usually very fit and well and on the advice of his GP has recently taken up swimming which he finds
helpful for a long-standing complaint of back pain. He is now swimming up to 30 lengths of the
swimming pool twice a week and feels well. The GP has performed some routine bloods examination
including FBC, electrolytes; these are normal. Current medication comprises amlodipine 5 mg daily and
lisinopril 20 mg daily. On examination pulse 70 bpm; blood pressure 200/100 mmHg; JVP not elevated;
S2 is normal; added S4. On auscultation in outpatients an ejection systolic murmur radiating to the
carotids is heard. There is an early diastolic murmur heard at the left sternal edge, and mid-to late-
diastolic murmur at the apex.
How would you assess clinically the severity of chronic aortic regurgitation in this patient if you establish
this patient has aortic regurgitation component? Choose the FALSE one.
a. Length of the diastolic murmur
b. Low diastolic pressure
c. Collapsing pulse
d. Degree of left ventricular dilatation
e. Fourth heart sound as clinical feature of heart failure
39. A 30 year-old bank executive is referred to outpatients for assessment by GP. He is known to be
hypertensive. On routine follow-up he has been found to have a systolic and diastolic murmur. He is
usually very fit and well and on the advice of his GP has recently taken up swimming which he finds
helpful for a long-standing complaint of back pain. He is now swimming up to 30 lengths of the
swimming pool twice a week and feels well. The GP has performed some routine bloods examination
including FBC, electrolytes; these are normal. Current medication comprises amlodipine 5 mg daily and
lisinopril 20 mg daily. On examination pulse 70 bpm; blood pressure 200/100 mmHg; JVP not elevated;
S2 is normal; added S4. On auscultation in outpatients an ejection systolic murmur radiating to the
carotids is heard. There is an early diastolic murmur heard at the left sternal edge, and mid-to late-
diastolic murmur at the apex.
An echocardiogram is performed and reported as follows: LV non-dilated; severe LVH; normal LV systolic
function; LA mildly dilated at 4.2 cm; aortic root 4.0 cm; aortic valve thickened (bicuspid?), aortic peak
gradient 20 mmHg, moderate AR; normal mitral valve; normal right heart.
Based on this echocardiogram interpretation, what is the most likely etiology of aortic regurgitation?
a. Coarctatio aorta
b. Endocarditis
c. Aortic dissection
d. Ankylosing spondylitis
e. Marfan's syndrome

40. A 30 year-old bank executive is referred to outpatients for assessment by GP. He is known to be
hypertensive. On routine follow-up he has been found to have a systolic and diastolic murmur. He is
usually very fit and well and on the advice of his GP has recently taken up swimming which he finds
helpful for a long-standing complaint of back pain. He is now swimming up to 30 lengths of the
swimming pool twice a week and feels well. The GP has performed some routine bloods examination
including FBC, electrolytes; these are normal. Current medication comprises amlodipine 5 mg daily and
lisinopril 20 mg daily. On examination pulse 70 bpm; blood pressure 200/100 mmHg; JVP not elevated;
S2 is normal; added S4. On auscultation in outpatients an ejection systolic murmur radiating to the
carotids is heard. There is an early diastolic murmur heard at the left sternal edge, and mid-to late-
diastolic murmur at the apex.
How would you assess clinically the severity of chronic aortic regurgitation in this patient if you establish
this patient has aortic regurgitation component? Choose the false one.
a. Collapsing pulse
b. Length of the diastolic murmur
c. Low diastolic pressure
d. Degree of left ventricular dilatation
e. Fourth heart sound as clinical feature of heart failure

41. A 30 year-old bank executive is referred to outpatients for assessment by GP. He is known to be
hypertensive. On routine follow-up he has been found to have a systolic and diastolic murmur. He is
usually very fit and well and on the advice of his GP has recently taken up swimming which he finds
helpful for a long-standing complaint of back pain. He is now swimming up to 30 lengths of the
swimming pool twice a week and feels well. The GP has performed some routine bloods examination
including FBC, electrolytes; these are normal. Current medication comprises amlodipine 5 mg daily and
Iisinopril 20 mg daily. On examination pulse 70 bpm; blood pressure 200/100 mmHg; JVP not elevated;
S2 is normal; added S4. On auscultation in outpatients an ejection systolic murmur radiating to the
carotids is heard. There is an early diastolic murmur heard at the left sternal edge, and mid-to late-
diastolic murmur at the apex.
An echocardiogram is performed and reported as follows: LV non-dilated; severe LVH; normal LV systolic
function; LA mildly dilated at 4.2 cm; aortic root 4.0 cm; aortic valve thickened (bicuspid?), aortic peak
gradient 20 mmHg, moderate AR; normal mitral valve; normal right heart.
Based on this echocardiogram interpretation, what is the most likely etiology of aortic regurgitation?
a. Marfan's syndrome
b. Aortic dissection
c. Coarctatto aorta
d. Ankylosing spondylitis
e. Endocarditis

42. A 30 year-old bank executive is referred to outpatients for assessment by GP. He is known to be
hypertensive. On routine follow-up he has been found to have a systolic and diastolic murmur. He is
usually very fit and well and on the advice of his GP has recently taken up swimming which he finds
helpful for a long-standing complaint of back pain. He is now swimming up to 30 lengths of the
swimming pool twice a week and feels well. The GP has performed some routine bloods examination
including FBC, electrolytes; these are normal. Current medication comprises amlodipine 5 mg daily and
Iisinopril 20 mg daily. On examination pulse 70 bpm; blood pressure 200/100 mmHg; JVP not elevated;
S2 is normal; added S4. On auscultation in outpatients an ejection systolic murmur radiating to the
carotids is heard. There is an early diastolic murmur heard at the left sternal edge, and mid-to late-
diastolic murmur at the apex.
Based on these findings there are some differential diagnosis can be established, and the most likely
diagnosis is?
a. Aortic regurgitation with ruptured sinus of Valsalva
b. Mixed aortic valve disease
c. Pure aortic regurgitation
d. Ruptured sinus of Valsalva
e. Aortic regurgitation with mitral stenosis
43. A 30 year-old bank executive is referred to outpatients for assessment by GP. He is known to be
hypertensive. On routine follow-up he has been found to have a systolic and diastolic murmur. He is
usually very fit and well and on the advice of his GP has recently taken up swimming which he finds
helpful for a long-standing complaint of back pain. He is now swimming up to 30 lengths of the
swimming pool twice a week and feels well. The GP has performed some routine bloods examination
including FBC, electrolytes; these are normal. Current medication comprises amlodipine 5 mg daily and
lisinopril 20 mg daily. On examination pulse 70 bpm; blood pressure 200/100 mmHg; JVP not elevated;
S2 is normal; added S4. On auscultation in outpatients an ejection systolic murmur radiating to the
carotids is heard. There is an early diastolic murmur heard at the left sternal edge, and mid-to late-
diastolic murmur at the apex.
All of the following are indications for surgery in pure aortic regurgitation, EXCEPT?
a. LV systolic dimension greater than 5 cm
b. Falling ejection fraction
c. Symptomatic AR
d. LV end diastolic dimension greater than 7 cm
e. Progressive LV dilation ((>0.75 cm over 12 months)

44. A 30 year-old bank executive is referred to outpatients for assessment by GP. He is known to be
hypertensive. On routine follow-up he has been found to have a systolic and diastolic murmur. He is
usually very fit and well and on the advice of his GP has recently taken up swimming which he finds
helpful for a long-standing complaint of back pain. He is now swimming up to 30 lengths of the
swimming pool twice a week and feels well. The GP has performed some routine bloods examination
including FBC, electrolytes; these are normal. Current medication comprises amlodipine 5 mg daily and
lisinopril 20 mg daily. On examination pulse 70 bpm; blood pressure 200/100 mmHg; JVP not elevated;
S2 is normal; added S4. On auscultation in outpatients an ejection systolic murmur radiating to the
carotids is heard. There is an early diastole murmur heard at the left sternal edge, and mid-to late-
diastolic murmur at the apex.
It is important to exclude endocarditis as a cause of this patient's deterioration. You would ask about of
the following EXCEPT?
a. Skin rashes
b. The history of hematuria
c. Weight loss, malaise or anorexia
d. Recent dental abscess
e. Non-hemorrhagic stroke previously

45. a 32-year-old man presents to establish care. His medical history is unremarkable. On physical
examination, the patient’s blood pressure is 132/84 mmHg, and his pulse is 88 beats/min. cardiac
examination reveals a soft systolic murmur, heard best at the left upper sternal border. The second
heart sound is split and has no variability with respirations. There is noevidence of cyanosis.
What is the most likely diagnosis for this patient?
a. Coronary arteriovenous fistula
b. Tetrallogy of Fallot
c. Pulmonary insufficiency
d. Bicuspid aortic valve
e. Ostium secundum defect 

46. a 32-year-old woman with primary PH is referred to follow up of her pulmonary pressures. In
addition of tricuspid regurgitant jet velocity, the following information is needed to estimate pulmonary
systolic pressure in this patient:
a. Antegrade velocity in the pulmonary artery
b. Hepatic vein flow
c. Mitral regurgitation maximal velocity
d. Imaging of the inferior vena cava
e. RV free wall thickness

47. a 35-year-old male presents to ED with 3 weeks of edema and progressive shortness of breath. BP
110/80 mmHg. HR 110 bpm, O2 sat 88%. Bedside echocardiogram shows LVEF 25%. Which of the
following therapies is most likely to acutely worsen the patient’s status?
a. Carvedilol 3.125 mg PO.
b. Captopril 12.5 mg PO.
c. Nitroglycerin 10 mcg/min IV gtt.
d. Furosemide 40 mg IV.

48. A 35-year-old man presents to your office for evaluation of valvular heart disease. He complains of
shortness of breath with only modest amounts of exertion, as well as two-pillow orthopnea. He also
complains of easy fatigability, as well as lower extremity edema and abdominal fullness. On
examination, he is in no acute distress. He is normotensive. Jugular venous pressure is elevated, with a
prominent a wave. The v wave is not easily discerned. S1 is loud. S2 is normal. A sound is heard in
diastole, 0.07 milliseconds after S2. A diastolic rumble is heard at the apex. A diastolic murmur is also
heard along the left sternal border, which increases with inspiration. Mild hepatomegaly is present.
There is 2+ peripheral edema.
What is your diagnosis?
a. Mitral stenosis and AS
b. Mitral stenosis with tricuspid insufficiency
c. Tricuspid stenosis
d. Mitral stenosis
e. Mitral and tricuspid stenosis

49. A 35-year-old man who underwent closure of an atrial septal defect at age 5 years was
asymptomatic and physically active until 3 months ago, when he began to have exertional dyspnea and
fatigue. He smokes one pack of cigarettes daily and drinks a six-pack of beer daily. He is a bricklayer and
had to stop working for the last 2 weeks. He takes no medications. On physical examination, blood
pressure is 105/80 mm Hg, and heart rate is 100/min, with an occasional extra systole. Jugular venous
pressure is 11 cm H20. The point of maximal impulse is displaced. The patient has a soft Si, a split S2,
and a grade 2/6 apical holosystolic murmur. The abdomen is distended, the liver is palpable 1 cm below
the right costal margin, and 2+ pedal edema is noted.
Laboratory tests show a total serum cholesterol of 180 mg/dL, serum thyroid-stimulating hormone of
2.5 ?U/mL, blood urea nitrogen of 32 mg/dL, serum creatinine of 1.3 mg/dL, alkaline phosphatase of 220
U/L, aspartate aminotransferase of 60 U/L, alanine aminotransferase of 75 U/L, and serum bilirubin of
1.2 mg/dL. Electrocardiogram shows nondiagnostic ST changes, with occasional premature ventricular
contractions.
Gated single-photon emission tomography scan shows an ejection fraction of 34%, with global
hyperkinesia.
Which of the following is the most likely cause of this patient’s heart failure?
a. Familial dilated cardiomyopathy
b. Coronary artery disease
c. Late heart failure as a result of repair of an atrial septal defect
d. Atrial septal defect patch dehiscence
e. Alcohol consumption 

50. A 35-year-old woman who emigrated to the United States is referred to you by her gynecologist for
evaluation of hypertension that was noted 1 week ago, when she sought an evaluation for infertility. She
was first told that she had hypertension at 20 years of age, but did not follow up with a physician until
recently. On your advice, her gynecologist initiated treatment with amlodipine, 5 mg, after obtaining a
blood pressure of 200/100 mm Hg. The patient has frequent headaches and also has cold feet and leg
cramping when she walks long distances.
Physical examination shows blood pressure of 160/90 mm Hg in the left arm while sitting and heart rate
of 70/min. Jugular venous pressure is normal. Carotid pulses are brisk bilaterally. Cardiac examination
shows a sustained apical impulse. SI is normal and S2 is physiologically split. An early systolic ejection
sound is noted, and an early peaking murmur is noted at the second right intercostal space. A short
diastolic murmur is audible along the left sternal border. Lungs are clear to auscultation.
Electrocardiogram shows left ventricular hypertrophy. Findings on urinalysis are normal.
Which of the following is the most appropriate next step in the evaluation of this patient?
a. measure the blood pressure in the lower extremities
b. Measure serum thyroid-stimulating hormone.
c. Order an echocardiogram.
d. Obtain a chest radiograph
e. Order a 24-hour urine test for metanephrine and vanillylmandelic acid.

51. A 38-year-old man came to emergency department with chief complaint shortness of breath, non-
radiating chest tightness and 2 sincopal episodes. The symptom has been felt since 1 month before
admission, but it was going worse in the recent 1 week. From the anamnesis, one week before
admission the patient felt pain and swelling on upper right leg, after he drove a car for two and a half
hours. The leg was then being massaged and the symptoms were going worse and he started feeling
shortness of breath. From the medical history, the patient was obese (Body Mass Index 31kg/m 2), and
has history of smoking for 19 years. The patient was a frequent distance traveler (average duration 4 to
6 hours for about 12 times/month). Patient has no history of hypertension and diabetes. On admission,
his blood pressure was 90/60 mmHg, pulse 120 times/minute and regular, respiratory rate 26
breaths/minute, temperature 36oc, and oxygen saturation 89%. Other physical examinations were
unremarkable.
Which of the following is the most likely diagnosis for this patient?
a. Acute Myocardial Infarction
b. Pulmonary Embolism
c. Aortic Dissection
d. Acute Lung Oedema
e. Cardiac Tamponade

52. A 38-year-old man came to emergency department with chief complaint shortness of breath, non-
radiating chest tightness and 2 sincopal episodes. The symptom has been felt since 1 month before
admission, but it was going worse in the recent 1 week. From the anamnesis, one week before
admission the patient felt pain and swelling on upper right leg, after he drove a car for two and a half
hours. The leg was then being massaged and the symptoms were going worse and he started feeling
shortness of breath. From the medical history, the patient was obese (Body Mass Index 31kg/m 2), and
has history of smoking for 19 years. The patient was a frequent distance traveler (average duration 4 to
6 hours for about 12 times/month). Patient has no history of hypertension and diabetes. On admission,
his blood pressure was 90/60 mmHg, pulse 120 times/minute and regular, respiratory rate 26
breaths/minute, temperature 36oc, and oxygen saturation 89%. Other physical examinations were
unremarkable.
What is the golden diagnostic tool for the above patient?
a. Chest CT Angiography
b. Coronary Angiography
c. Cardiac Marker
d. Electrocardiography
e. Echocardiography

53. A 38-year-old man came to emergency department with chief complaint shortness of breath, non-
radiating chest tightness and 2 sincopal episodes. The symptom has been felt since 1 month before
admission, but it was going worse in the recent 1 week. From the anamnesis, one week before
admission the patient felt pain and swelling on upper right leg, after he drove a car for two and a half
hours. The leg was then being massaged and the symptoms were going worse and he started feeling
shortness of breath. From the medical history, the patient was obese (Body Mass Index 31kg/m 2), and
has history of smoking for 19 years. The patient was a frequent distance traveler (average duration 4 to
6 hours for about 12 times/month). Patient has no history of hypertension and diabetes. On admission,
his blood pressure was 90/60 mmHg, pulse 120 times/minute and regular, respiratory rate 26
breaths/minute, temperature 36oc, and oxygen saturation 89%. Other physical examinations were
unremarkable.
Which of the following echocardiographic findings that can be found in above patients?
a. RV acceleration time >60 ms in the presence of tricuspid insufficiency pressure gradient >60
mmHg
b. Acute mitral regurgitation
c. Hypokinesis of the anterior and inferior wall
d. Right ventricular free wall hypokinesis in the presence of normal right ventricular apical.
e. Visualization of true lumen and false lumen
54. A 38-year-old man came to emergency department with chief complaint shortness of breath, non-
radiating chest tightness and 2 sincopal episodes. The symptom has been felt since 1 month before
admission, but it was going worse in the recent 1 week. From the anamnesis, one week before
admission the patient felt pain and swelling on upper right leg, after he drove a car for two and a half
hours. The leg was then being massaged and the symptoms were going worse and he started feeling
shortness of breath. From the medical history, the patient was obese (Body Mass Index 31kg/m 2), and
has history of smoking for 19 years. The patient was a frequent distance traveler (average duration 4 to
6 hours for about 12 times/month). Patient has no history of hypertension and diabetes. On admission,
his blood pressure was 90/60 mmHg, pulse 120 times/minute and regular, respiratory rate 26
breaths/minute, temperature 36oc, and oxygen saturation 89%. Other physical examinations were
unremarkable.
Below is ECG findings that can be found in the above patient, EXCEPT:
a. ST elevation with Right Bundle Branch Block in VI-V2
b. Sinus Tachycardia
c. Left Bundle branch Block
d. Atrial arrhythmia with P pulmonale
e. Inverted T wave in anterior lead

55. A 38-year-old man came to emergency department with chief complaint shortness of breath, non-
radiating chest tightness and 2 sincopal episodes. The symptom has been felt since 1 month before
admission, but it was going worse in the recent 1 week. From the anamnesis, one week before
admission the patient felt pain and swelling on upper right leg, after he drove a car for two and a half
hours. The leg was then being massaged and the symptoms were going worse and he started feeling
shortness of breath. From the medical history, the patient was obese (Body Mass Index 31kg/m 2), and
has history of smoking for 19 years. The patient was a frequent distance traveler (average duration 4 to
6 hours for about 12 times/month). Patient has no history of hypertension and diabetes. On admission,
his blood pressure was 90/60 mmHg, pulse 120 times/minute and regular, respiratory rate 26
breaths/minute, temperature 36oc, and oxygen saturation 89%. Other physical examinations were
unremarkable.
What is the most important therapy for above patients?
a. Primary Percutaneous Intervention
b. Urgent surgery
c. Low Mollecular Weight Heparin
d. Pericardiocentesis
e. Fibrinolysis

56. A 40-year-old asymptomatic man with no risk factors undergoes stress testing as part of an
"Executive Physical" program. His resting ECG is normal and he is taking no medications. He has an
exercise capacity of 14 METs (13.5 minutes on the Bruce protocol), no angina, a peak heart rate of 180,
and 1 mm of down-sloping ST-segment depression noted in lead V5. His Duke treadmill score is
a. 7.5
b. 3.5
c. 2
d. 9
e. 8.5

57. A 40-year-old asymptomatic man with no risk factors undergoes stress testing as part of an
"Executive Physical" program. His resting ECG is normal and he is taking no medications. He has an
exercise capacity of 14 METs (13.5 minutes on the Bruce protocol), no angina, a peak heart rate of 180,
and 1 mm of down-sloping ST-segment depression noted in lead V5.
Given these test results, the next most appropriate step is
a. Coronary angiography
b. A repeat stress test in 1 year
c. A stress imaging study
d. No further cardiac testing

58. A 43-year-old man comes to the emergency room for evaluation of severe chest pain that awoke him
from sleep. While the patient was seated, the pain gradually resolved after 5 minutes, but recurred
several minutes later. The patient reports that he has had nasal congestion, nausea, fatigue, and a tow-
grade fever for the past few days. Physical examination shows a temperature of 38 °C (100.4 °F), heart
rate of 104/min, and recurrence of pain while lying in the left lateral decubitus position. A three-
component pericardial friction rub is noted. Laboratory findings include a total leukocyte count of
11,000/?L, an erythrocyte sedimentation rate of 55 mm/h, and a slightly elevated troponin level.
Electrocardiogram shows diffuse ST-T-wave changes
Which of the following is the best course of treatment for this patient?
a. Colchicine, 0.5 mg twice a day orally for 2 weeks
b. Metoprolol, 25 mg twice a day orally
c. Prednisone, 25 mg/d orally for 2 weeks, followed by a taper
d. Reteplase, 10 U by intravenous bolus, repeated in 30 minutes
e. Indomethacin, 25 mg four times a day orally for 2 weeks

59. A 45-year old business executive presents to A&E with a 2-hour history of central crushing chest pain
and breathlessness. He is a non-smoker, previously very fit and well and attends a gym four times a
week. There is no family history of ischemic heart disease. His cholesterol measured at an insurance
medical was 3,3 mmol/L. His observations on admission are as follows; pulse 105 bpm; blood pressure
80/50 mmHg; 02 saturations 90% on room air. He is apyrexial. An ECG is performed and shown sinus
tachycardia of 105 bpm. Right-axis deviation and non-specific T-wave inversion in leads III, aVF, V2-V4.
This patient returned to his country from a business trip in Australia 2 weeks previously.
On clinical examination, you can find all of the following, EXCEPT?
a. Jugular venous pressure is elevated
b. There is a soft third heart sound over the the right sternal border
c. The troponin I within normal limits (< 1 ug/L)
d. Cyanosed and cool peripherally
e. The D-dimer is elevated to the 5 times the normal range
60. A 45-year old business executive presents to A&E with a 2-hour history of central crushing chest pain
and breathlessness. He is a non-smoker, previously very fit and well and attends a gym four times a
week. There is no family history of ischemic heart disease. His cholesterol measured at an insurance
medical was 127 mg/L. His observations on admission are as follows; pulse 105 bpm; blood pressure
80/50 mmHg; 02 saturations 90% on room air. He is apyrexial. An ECG is performed and shown sinus
tachycardia of 105 bpm. Right-axis deviation and non-specific T-wave inversion in leads III, aVF, V2-V4.
For immediate management you can perform all of the following, EXCEPT?
a. High flow oxygen
b. Opiates should be avoided if possible
c. Airway breathing and circulation is required
d. Large volume of intravenous fluids
e. The patients should be nursed lying flat or with a head-down tilt 

61. A 45-year old business executive presents to A&E with a 2-hour history of central crushing chest pain
and breathlessness. He is a non-smoker, previously very fit and well and attends a gym four times a
week. There is no family history of ischemic heart disease. His cholesterol measured at an insurance
medical was 127 mg/L. His observations on admission are as follows; pulse 105 bpm; blood pressure
80/50 mmHg; 02 saturations 90% on room air. He is apyrexial. An ECG is performed and shown sinus
tachycardia of 105 bpm. Right-axis deviation and non-specific T-wave inversion in leads III, aVF, V2-V4.
The patient responds to the intravenous fluid replacement (blood pressure 100/70 mmHg, pulse 100
bpm) but remains hypoxic (02 saturation 90%).
What further imaging may be helpful at this point?
a. Chest X-ray is enough to confirm pulmonary embolic disease
b. Transthoracic echocardiogram can be very useful diagnostically
c. Perfusion scan can help to exclude pulmonary hypertension
d. Ventilation perfusion scan can identifying chronic cor pulmonale
e. CT pulmonary angiogram has a moderate sensitivity for acute cor pulmonale

62. A 45 year-old man is admitted to a hospital due to typical chest pain after exercise with 4 hours
onset. He reveals that this not the first time, the symptoms were already develop over one year and he
had a syncope last month. The patient was a heavy smoker, his father already passed away at his 50's
due to heart disease. On physical examination the blood pressure is 140/80 mmHg, normal SI and S2
with a grade 3/6 holosystolic murmur at the apex and axilla. An Electrocardiogram shows left ventricular
hypertrophy with strain, left atrial enlargement. Cardiac enzyme were normal.
What is the best modalities to define the diagnosis of this patient?
a. Transesophageal Echocardiogray
b. Cardiac MRI
c. Transthoracic echocardiography
d. Coronary Angiography
e. Cardiac CT

63. A 45 year-old man is admitted to a hospital due to typical chest pain after exercise with 4 hours
onset. He reveals that this not the first time, the symptoms were already develop over one year and he
had a syncope last month. The patient was a heavy smoker, his father already passed away at his 50's
due to heart disease. On physical examination the blood pressure is 140/80 mmHg, normal SI and S2
with a grade 3/6 holosystolic murmur at the apex and axilla. An Electrocardiogram shows left ventricular
hypertrophy with strain, left atrial enlargement. Cardiac enzyme were normal.
This following were the major clinical features associated with increased risk of SCD that could possibly
happen to this patient, EXCEPT?
a. Age
b. Frequent syncope episode
c. Family History
d. Non-sustained ventricular tachycardia
e. Gender 
64. A 45 year-old man is admitted to a hospital due to typical chest pain after exercise with 4 hours
onset. He reveals that this not the first time, the symptoms were already develop over one year and he
had a syncope last month. The patient was a heavy smoker, his father already passed away at his 50's
due to heart disease. On physical examination the blood pressure is 140/80 mmHg, normal SI and S2
with a grade 3/6 holosystolic murmur at the apex and axilla. An Electrocardiogram shows left ventricular
hyperthropy with strain, left atrial enlargement. Cardiac enzyme were normal.
This patient could possibly having :
a. Aortic Stenosis
b. Farby Disease
c. Hypertensive Heart Disease
d. Hypertrophic Cardiomyopathy
e. Amyloidosis 

65. A 45 year-old man is admitted to a hospital due to typical chest pain after exercise with 4 hours
onset. He reveals that this not the first time, the symptoms were already develop over one year and he
had a syncope last month. The patient was a heavy smoker, his father already passed away at his 50's
due to heart disease. On physical examination the blood pressure is 140/80 mmHg, normal SI and S2
with a grade 3/6 holosystolic murmur at the apex and axilla. An Electrocardiogram shows left ventricular
hypertrophy with strain, left atrial enlargement. Cardiac enzyme were normal.
On imaging studies shows this following findings, EXCEPT?
a. Pulmonary venous systolic flow reversal
b. LVOT pressure gradient 35mmHg
c. Systolic anterior motion of the mitral leaflet
d. LV wall thickness is 32mm symmetrical 

66. A 45-year-old man with rheumatic mitral stenosis presents for further evaluation. In the past 2 to 3
years, he has noted progressive dyspnea with less than moderate activity. He was started on a ?-blocker
1 year ago, but remains symptomatic. Echocardiogram reveals a mean mitral gradient of 4 mmHg with a
valve area of 1.6 cm2. As there was a discrepancy between the degree of symptoms and resting
hemodynamics you proceed to a stress echocardiogram that revealed a post stress PA pressure of 70
mmHg and a mean transmitral gradient of 17 mmHg. You decide to send this patient for percutaneous
intervention.
What is the most appropriate test to order at the time of or prior to the valvuloplasty procedure?
a. Stress nuclear perfusion study
b. 24-Hour electrocardiographic monitoring to assess for paroxysmal atrial fibrillation
c. Cardiac CT to assess for aortic calcification 
d. Thransesophageal echocardiogram

67. A 45-year-old music teacher with a history of bicuspid aortic valve status post aortic valve
replacement is admitted to the hospital with progressive fatigue and dyspnea. She admits to
nonadherence with sodium restriction, and has been drinking 2-3 glasses of wine per night. Her bedside
hemodynamic evaluation suggests elevated right and left heart filling pressures and low cardiac output.
An echocardiogram reveals moderate LV dilation with an end-diastolic dimension of 6.4 cm, moderate
systolic dsyfunction with an EF of 30%, a well-seated mechanical aortic valve with normal leaflet
function, 3+ mitral regurgitation, and 2+ tricuspid regurgitation.
She responds well to empiric treatment with IV diuretics for 3 days, with resolution of dyspnea. Her
examination then reveals jugular venous pressure of 6 cm of water, no hepatomegaly and warm
extremities without edema. Plans are made for discharge home with follow-up in the HF clinic in 1 week.
Which of the following is a quality measure recommended by the ACCF/AHA for all HF patients at the
time of hospital discharge?
a. Counseling regarding alcohol cessation.
b. Documentation of LV size.
c. Optimizing therapy with an aldosterone antagonist.
d. Achieving optimal volume status
e. Scheduling a follow-up visit within 1 week.

68. A 45 year old woman is evaluated for palpitations that occur intermittently during the day, vary in
severity, and cause a sensation of skipped beats. She has no other associated symptoms. She had a
similar episode 2 years ago while undergoing a stressful job relocation, but did not seek medical
attention at that time. She is now under pressure at work, and her son is leaving for college in 1 week.
On physical examination, her blood pressure is 160/90 mm Hg and her heart rate is 80/min. Cardiac
examination shows normal heart sounds and no murmurs. Electrocardiogram shows sinus rhythm with
premature atrial contractions, and a 24-hour ambulatory monitor shows 5673 premature atrial
contractions, 127 premature ventricular contractions, and no runs of arrhythmias. Results of laboratory
tests, including thyroid function tests and complete blood count, are normal. The patient remains highly
symptomatic, despite reassurance.
Which of the following is the most appropriate next step in the management of this patient?
a. Perform an electrophysiology study
b. Perform an exercise treadmill test
c. Start disopyramide therapy
d. Start diuretic therapy
e. Start beta-blocker therapy

69. A 45 year old woman presents to the emergency department with eight hours of chest pain. The
pain is constant, severe, and midsternal in location. She notes that it is worse when she lies down. She
denies exposure to sick friends or relatives. Past history is notable for tobacco use, borderline
hypertension, and elevated cholesterol. Her father died of a myocardial infarction at age 67 years.
Examination is notable for a heart rate of 104, and a blood pressure of 125/80 in both arms. Her lungs
are clear. On cardiac examination, she has a prominent friction rub with two out of three components
present. The remainder of the examination is unremarkable.
Which of the following ECG abnormalities does not suggest pericarditis in the absence of an effusion?
a. PR segment elevation in lead aVR
b. Sinus tachycardia
c. PR segment depressbn
d. Electrical alternans
e. Concave upward ST segment elevation

70. A 45 year old woman presents to the emergency department with eight hours of chest pain. The
pain is constant, severe, and midsternal in location. She notes that it is worse when she lies down. She
denies exposure to sick friends or relatives. Past history is notable for tobacco use, borderline
hypertension, and elevated cholesterol. Her father died of a myocardial infarction at age 67 years.
Examination is notable for a heart rate of 104, and a blood pressure of 125/80 in both arms. Her lungs
are clear. On cardiac examination, she has a prominent friction rub with two out of three components
present. The remainder of the examination is unremarkable.
Which of the following ECG abnormalities does not suggest pericarditis in the absence of an effusion?
a. Sinus tachycardia
b. PR segment elevation in lead aVR
c. PR segment depression
d. Concave upward ST segment elevation
e. Electric alternans

71. A 46-year-old man begins an exercise training program that involves jogging on a treadmill for 40
minutes three times per week. Before beginning the program, he underwent exercise testing that
showed a peak heart rate of 120/min and a peak estimated metabolic equivalent level of 7. After 6
months of training, he undergoes another treadmill test that shows a peak heart rate of 120/min and a
peak metabolic equivalent level of 9. Both tests were terminated because of patient fatigue. He had no
ischemic responses on either test.
Which of the following best describes the results of this training program?
a. The findings indicate that the patient did not exercise to exhaustion during the first test.
b. Changes in the peak metabolic equivalent level are most likely caused by changes in
pulmonary ventilation that occur with exercise training.
c. The finding of no change in peak heart rate indicates that no training benefit was achieved.
d. Achievement of a higher peak metabolic equivalent level is primarily caused by a greater level
of effort on the second test.
e. The heart rate response to submaximal levels of exercise is now lower.

72. A 46-year-old man with mitral stenosis presents for evaluation. He has NYHA class II-III shortness of
breath.
Physical Examination is consistent and echo cardiogram confirmed the diagnosis of severe mitral
stenosis.
He undergoes percutaneous valvuloplasty. The following morning, on examination, you note that he is
comfortable. His oxygen saturation is 100% on room air. Opening snap is 0.12 milliseconds after S2. A
shorter decrescendo diastolic rumble is heard. You obtain a predischarge echocardiogram.
The echocardiogram reveals a small left-to-right shunt at the atrial level by color. What do you
recommend?
a. Observation
b. Referral for surgical closure
c. Indefinite anticoagulation
d. Referral for percutaneous closure

73. A 46-year-old woman with history of breast cancer in remission presents with a 2-week history of
dyspnea on exertion and weight gain. She does not have any history of cardiac disease, but has a history
of hypertension and type 2 diabetes mellitus. She presents to the emergency room because she now has
shortness of breath at rest. She denies other cardiac complaints.
On admission, her creatinine is 1.2 mg/dl, potassium is 4.4 mEq/L, HbA1c is 6.4%, blood glucose is 123
mg/dl, low-density lipoprotein (LDL) 112, high-density lipoprotein (HDL) 32, triglycerides 188 mg/dl, and
BNP is 2202 pg/ml. Her ECG shows sinus tachycardia, 110 bpm, intraventricular conduction delay with a
prolonged QRS interval of 128 ms.
After intravenous diuretic therapy in the emergency room, she gets admitted and an echocardiogram
reveals a dilated LV of 7 cm, and LVEF of 10% with spontaneous echocontrast in the LV. On day 3, she
undergoes coronary angiography, which reveals angiographically normal coronary arteries. An ACE
inhibitor and a beta-blocker are started during the hospital course, and she tolerates these well with a
modest improvement in her symptoms.
Which of the following is the best additional treatment choice at this point?
a. Statin.
b. Aldosterone antagonist
c. Aspirin.
d. Cardiac resynchronization therapy with defibrillator.
e. Cardiac resynchronization therapy without defibrillator.

74. A 50-year-old African-American man with no significant past medical history presents with
palpitations. On examination, he has a mildly elevated jugular venous pressure at 10 cm H20, a faint
parasternal lift, and an RV S3 gallop. His ECG is notable for sinus rhythm, with incomplete right bundle
branch block. A Holter monitor reveals frequent runs of nonsustained ventricular tachycardia with a left
bundle branch block pattern. Coronary angiography reveals no significant epicardial coronary artery
disease. Echocardiogram suggests RV cavity dilation and moderate tricuspid valve regurgitation.
This patient most likely has a disorder associated with a defect in the gene encoding which of the
following proteins?
a. Dystrophin.
b. Plakoglobin
c. Lamin A/C.
d. Tafazzin.
e. Myosin heavy chain.

75. A 50 years old male patient brough this treadmill test result that performed after he had finished
phase II of cardiac rehabilitation program after undergone elective PCI. His baseline HR was 70bpm.
There was no ST-T changes, no arrhythmia, normal response of BP and HR during exercise. The test was
stopped at 9 minutes of exercise (Bruce protocol), with HR 150 (Rate Perceived Exertion : Borg Scale 17).
What is his HR target for treadmill at home using 65-75% HRR (Heart Rate Reserve) formula?
a. 122 - 130bpm
b. 102- 107bpm
c. 97 - 112bpm
d. Can't be measured
e. 110- 127bpm

76. A 50-year-old-man had a 3 graft CABG operation, LIMA to LAD, SVG to distal LCX, and SVG to distal
RCA with Aox time 90 minutes and CPB time 120 minutes. ECG pre-op showed normal and went well
during operation. EF pre-op was 67% with global normokinetic. On day 0 at ICU, the hemodynamic was
unstable, Lactate was 10, with mixed vein 45. After 3 days in ICU patient stable and plan move to
Intermediate wards but before move patient had atrial fibrillation.
What kind of medication that should be given to a post-op CABG patient to prevent atrial fibrillation?
a. All the answers are correct
b. Ace-inhibitor
c. Aspilet
d. Statin
e. B-blocker

77. A 50-year-old-man had a 3 graft CABG operation, LIMA to LAD, SVG to distal LCX, and SVG to distal
RCA with Aox time 90 minutes and CPB time 120 minutes. ECG pre-op showed normal and went well
during operation. EF pre-op was 67% with global normokinetic. On day 0 at ICU, the hemodynamic was
unstable, Lactate was 10, with mixed vein 45. After 3 days in ICU patient stable and plan move to
Intermediate wards but before move patient had atrial fibrillation.
When should B-blocker be given to a post-op CABG patient to prevent atrial fibrillation?
a. While the patient on ward just after rehabilitation
b. While the patient move to IW
c. Day 1
d. Day 0
e. As soon as possible after extubation

78. A 50-year-old-man had a 3 graft CABG operation, LIMA to LAD, SVG to distal LCX, and SVG to distal
RCA with Aox time 90 minutes and CPB time 120 minutes. ECG pre-op showed normal and went well
during operation. EF pre-op was 67% with global normokinetic. On day 0 at ICU, the hemodynamic was
unstable, Lactate was 10, with mixed vein 45. After 3 days in ICU patient stable and plan move to
Intermediate wards but before move patient had atrial fibrillation.
Choose one of the criteria that should be considered if he had PMI?
a. None above are correct
b. Signs of tamponade
c. NRWMA was negative
d. Less volume
e. Unstable hemodynamic if the other causes were ignored

79. A 50-year-old-man had a 3 graft CABG operation, LIMA to LAD, SVG to distal LCX, and SVG to distal
RCA with Aox time 90 minutes and CPB time 120 minutes. ECG pre-op showed normal and went well
during operation. EF pre-op was 67% with global normokinetic. On day 0 at ICU, the hemodynamic was
unstable, Lactate was 10, with mixed vein 45. After 3 days in ICU patient stable and plan move to
Intermediate wards but before move patient had atrial fibrillation.
What are the possibilities happened on the patient on day 0 post-op that made he had unstable
hemodynamic?
a. Bleeding
b. All the answers are correct
c. Perioperative myocardial infarction
d. Acute left heart failure
e. Volume depletion 

80. A 50 year old woman has left shoulder discomfort that she notices when she climbs stairs, walks
uphill, or becomes upset with her children. The discomfort generally resolves within one or two minutes
after she stops the activity. Her resting ECG showed nonspecific ST- and T-wave abnormalities, with less
than 1mm of ST-segment depression. The patient had a TMET and exercised for 6 minutes on a Bruce
protocol to a HR of 130 bpm and a BP of 155/70 mmHg. She stopped because of severe chest heaviness
and left shoulder pain. The exercise ECG did not show any ST-segment depression. Which of the
following statements is correct about this patient's Duke treadmill score?
a. It is not clinically meaningful because the treadmill score applies only to men
b. It is calculated as -2 and places the patient at intermediate risk for subsequent cardiac event
c. It is calculated as -2 and places the patient at low-to-intermediate risk for subsequent cardiac
events
d. It is not clinically meaningful because the treadmill score applies only to patients with normal
resting ECGs
e. It is calculated as +6 and places the patient at low risk for subsequent cardiac events

81. A 52-year-old woman is referred for shortness of breath. Her clinical examination shows a 2/6
diastolic murmur along the LSB and a wide pulse pressure. The patient has no signs of heart failure but
has a third heart sound and a soft systolic murmur of MR.
In patients with a barely audible diastolic murmur and heart failure, what sign is suggestive that severe
AR is the cause of the heart failure?
a. A murmur of functional MR
b. A decreased first heart sound
c. A BP of 130/45 mmHg
d. A third heart sound
e. An increased second heart sound

82. A 55-year-old male presents with shortness of breath for 6 months. He's been sleeping in a chair for
the past month because he coughs whenever he lies back. He's also noticed that his heart is racing most
of the time. He's had no antecedent illnesses. He tells you that his brother died suddenly several years
ago in his 30s. He drinks socially.
He appears comfortable with a BP of 110/75 and a regular HR of 115. His jugular venous pressure is 18
cm of water, lungs are clear, and the heart is enlarged on palpation. He has a gallop rhythm and a diffuse
precordial systolic murmur. The abdomen is mildly tender in the RUQ with a liver edge easily palpable.
He is warm with trace pedal edema.
An EKG demonstrates sinus tachycardia, and narrow QRS.
In addition to diuretic management, you initiate:
a. Digoxin.
b. Aldosterone antagonist.
c. Beta-blocker.
d. Angiotensin receptor blocker (ARB).
e. Angiotensin converting enzyme (ACE) inhibitor.

83. A 55-year-old woman is hospitalized because of orthopnea and paroxysmal nocturnal dyspnea.
Echocardiography shows an ejection fraction of 20%, and a diagnosis of dilated cardiomyopathy is made.
Diuretic treatment is initiated, beginning with furosemide administered intravenously and followed by
furosemide, 40 mg twice a day orally. Treatment with an ACE inhibitor is also initiated. On admission,
the blood urea nitrogen level is 25 mg/dL and serum creatinine level is 1 .6 mg/dL. After the dose of
captopril is increased from 6.25 mg three times a day to 12.5 mg three times a day on hospital day 2, the
blood urea nitrogen level is 42 mg/dL, creatinine level is 2.0 mg/dL, and potassium level is 4.6 meg/L.
Net diuresis in the hospital is 2.4 L over 36 hours. On physical examination, blood pressure is 105/85 mm
Hg, and heart rate is 88/min. The crackles have resolved, and jugular venous pressure is normal. Trace
pedal edema is noted.
Which of the following is the most appropriate next step in the management of this patient?
a. Decrease the dosage of captopril
b. Start treatment with low-dose dobutamine
c. Administer a fluid bolus
d. Discontinue captopril
e. Decrease the dosage of the diuretic

84. A 55-year-old woman with a history of unrepaired ventricular septal defect (VSD) and Eisenmenger
physiology presents to your clinic. Her most recent hemoglobin measured 17.5 g/dL (11.5 to 15.0),
hematocrit 55% (36 to 46), mean corpuscular volume (MCV) 76 (83 to 99), and platelet count 140 x 103
cells/?L (150 to 400). She reports New York Heart Association class m limitations with increasing fatigue
and infrequent headaches over the last 6 months. Blood pressure (BP) 110/70 mmHg, heart rate (HR) 70
BPM, and regular, 02 saturation 77% on room air. Cardiac examination reveals peripheral clubbing and
cyanosis, a parasternal heave, bud P2, and a murmur of tricuspid regurgitation (TR). Electrocardiogram
(ECG) shows sinus rhythm. She has had repeat phlebotomy for treatment of presumed hyperviscosity
syndrome but does not feel any better.
What of the following options is the most appropriate next step in management?
a. Phlebotomy should continue until symptoms improve.
b. Measure serum erythropoietin
c. Obtain iron studies.
d. Commence a pulmonary vasodilator.
e. Refer for heart-lung transplantation.

85. A 56-year-old man with mitral stenosis presents for evaluation. He has NYHA class II-III shortness of
breath.
Physical Examination
-. He is in no acute distress.
-. JVP is mildly elevated.
-. Pulse is regular at 80 bpm.
-. Chest is clear.
-. Cardiac: Nondisplaced PMI. Opening snap heard 0.09 milliseconds after S2. Long diastolic rumble. No
peripheral edema.
Echocardiogram reveals a planimetered mitral valve area of 1.2 cm2. Mean gradient 10 mmHg. Pressure
half-time of 185 milliseconds.
He undergoes percutaneous valvuloplasty. The following morning, on examination, you note that he is
comfortable. His oxygen saturation is 100% on room air. Opening snap is 0.12 milliseconds after S2. A
shorter decrescendo diastolic rumble is heard. You obtain a predischarge echocardiogram. The report
indicates a pressure half-time of 180 milliseconds.
What do you do next based on the echocardiogram?
a. There was a less-than-optimal result from the valvuloplasty. No significant change in mitral
valve area was achieved. You plan to send him for another procedure or surgery.
b. There was an error in half-time measurement. You order a repeat assessment of pressure
half-time later that day.
c. Repeat echocardiogram with planimetry of mitral valve area.
d. Consider TEE to see the valve opening better

86. A 56-year-old woman who is new to your practice is evaluated for recent exacerbation of dyspnea
and fatigue. She has idiopathic dilated cardiomyopathy and receives a stable heart failure regimen,
including lisinopril, 20 mg/d; digoxin, 125 mg/d; furosemide, 40 mg/d; and metoprolol XL, 50 mg/d. She
also takes alendronate, hormone replacement therapy, glipizide, folic acid, and ibuprofen because of
rheumatoid arthritis. Thyroid hormone suppressive therapy with thyroxin was initiated because of the
finding of an elevated serum thyroid-stimulating hormone level 4 months earlier. The thyroid-
stimulating hormone level returned to normal after therapy.
On physical examination, blood pressure is 110/72 mm Hg, and heart rate is 82/min. Jugular venous
pressure is estimated at 10 cm H20. The lungs are clear. Cardiac examination shows an S3 gallop and 2+
pitting edema.
Which of the following is most likely to be causing the exacerbation of congestive heart failure in this
patient?
a. Glipizide
b. Ibuprofen
c. Thyroxin
d. Alendronate
e. Estrogen

87. A 58-year-old man presents with atypical chest pain. Coronary angiography reveals a 60% diameter
stenosis in the mid-LAD coronary artery.
Which of the following intracoronary ultrasound or Doppler measurements is the most sensitive for
determining if this intermediate stenosis is hemodynamically significant?
a. Proximal-to-distal velocity ratio
b. Luminal are
c. Coronary flow reserve
d. Absolute coronary flow
e. Percent plaque area in the stenosis 

88. A 59-year-old man presents for further evaluation of recurrent congestive heart failure. He appears
to be in no acute distress on your evaluation. BP is 100/60 mmHg. Carotid upstrokes are weak, but not
delayed. Chest examination shows minimal bibasilar rales. PMI is displaced and sustained. A summation
gallop is present. There is an increased P2. There is mild peripheral edema. An echocardiogram reveals a
dilated LV with an ejection fraction of 25%. The aortic valve does have some calcification, with restricted
leaflet excursion. Peak/mean gradients are 25/15 mmHg. By the continuity equation, the aortic valve
area is calculated as 0.7 cm2.
What is your next step?
a. Referral for cardiac transplant
b. Dobutamine echocardiogram
c. Immediate referral for aortic valve replacement (AVR)
c. Start an ACEI 

89. A 60 yo man presented to the cardiovascular clinic for consultation. He described typical angina
climbing one flight of stairs. Symptom was improved with ISDN 5 mg si. He was smoker and had history
of hypertension for 5 yrs which was controlled with amlodipine 5 mg od. Resting ECG was normal. Which
of the following statements about the case is correct?
a. Coronary angiography is indicated because high risk patient
b. Coronary angiography is Inappropriate because the patient is stable and well controlled with
medicamentosa c. Coronary angiography is appropriate for the patient without prior noninvasive
stress test
d. Risk stratification with noninvasive stress test has to be done before refers the patient to
cath lab for coronary angiography

90. A 60-year-old man presents to the emergency room with complaints of weakness, lethargy, and
severe dyspnea. One week prior, his family notes that he complained of chest pressure that lasted for
several hours. On physical examination, he appears to be in respiratory distress. Blood pressure (BP) is
80/50 mmHg. Heart rate is 130 bpm. His oxygen saturation is 87% on room air. Chest examination
reveals diffuse crackles. Cardiac examination reveals a nondisplaced point of maximum impulse (PMI).
Third and fourth heart sounds are heard, as is an apical systolic murmur. No thrill is present.
Electrocardiogram reveals inferior Q waves without ST-segment elevation. He is urgently intubated and
pressors are started. An intra-aortic balloon pump is placed. A surface echocardiogram reveals a normal-
sized left atrium and a mild jet of mitral regurgitation (MR).
What test do you perform first?
a. Administration of thrombolytic therapy
b. Right heart catheterization with an oxygen saturation run
c. Transesophageal echocardiography (TEE)
d. Cardiac catheterization

91. A 60-year-old man presents to the emergency room with complaints of weakness, lethargy, and
severe dyspnea. One week prior, his family notes that he complained of chest pressure that lasted for
several hours. On physical examination, he appears to be in respiratory distress. Blood pressure (BP) is
80/50 mmHg. Heart rate is 130 bpm. His oxygen saturation is 87% on room air. Chest examination
reveals diffuse crackles. Cardiac examination reveals a nondisplaced point of maximum impulse (PMI).
Third and fourth heart sounds are heard, as is an apical systolic murmur. No thrill is present.
Electrocardiogram reveals inferior Q waves without ST- segment elevation. He is urgently intubated and
pressors are started. An intra-aortic balloon pump is placed. A surface echocardiogram reveals a normal-
sized left atrium and a mild jet of mitral regurgitation (MR).
What test do you perform first?
a. Cardiac catheterization
b. Right heart catheterization with an oxygen saturation run
c. Transesophageal echocardiography (TEE)
d. Administration of thrombolytic therapy

92. a 60 year old man with AF was started on warfarin 8 months ago. He present to the emergency room
with severe bleeding. Blood work levels that INR of 4.9. hemoglobin 12 mg/dL and platelet count
200.000. you decide to immediately reverse his warfarin with?
a. Thrombocyte transfusion
b. Dyalisis
c. Vitamin K
d. Cryoprecipitate

93. A 62-year-old man with a history of rheumatic heart disease presents to your office with complaints
of exertional dyspnea. No constitutional complaints are present. He had undergone a mitral valve
replacement with a bileaflet tilting disk mechanical valve 11 years prior. He is normotensive with a heart
rate of 73 bpm. On examination, you note a grade n/VI holosystolic murmur at the apex. An
echocardiogram is performed, which reveals normal LV and RV function. Peak mitral gradient is 30
mmHg. Mean transmitral gradient is 7 mmHg. Pressure half-time is 80 milliseconds.
What is your next diagnostic step?
a. Invasive assessment of hemodynamics
b. Transesophageal echocardiogram
c. Fluoroscopy of the valve
e. Draw blood cultures

94. A 63-year-old man is evaluated for a rash. During evaluation, you note a grade 3/6 holosystolic
murmur at the apex that radiates throughout the precordium and is loudest along the left sternal
border. The left ventricular point of maximum impulse is diffuse and displaced laterally. The heart rate is
94/min and blood pressure is 136/80 mm Hg. Jugular venous pressure is increased. The rest of the
findings on physical examination are unremarkable, except for an area of contact dermatitis under the
patient's wristwatch band. The patient works full-time as a dairy farmer and has not seen a physician for
10 years. He denies shortness of breath, orthopnea, or paroxysmal nocturnal dyspnea.
Electrocardiogram shows sinus rhythm with left atrial enlargement. Echocardiography shows
myxomatous degeneration of the mitral valve, with partial flail of the posterior leaflet and severe mitral
regurgitation. The left atrium is enlarged. The left ventricle is also enlarged, measuring 71 mm in diastole
(normal range, 37 to 57 mm) and 51 mm in systole (normal range for fractional shortening, 28 mm to 44
mm). The left ventricular ejection fraction is estimated at 52% (normal ?55%).
Which of the following statements about surgical intervention for this patient is true?
a. Surgery is indicated if transesophageal echocardiographic imaging shows that the mitral valve
is amenable to repair,
b. Surgery is indicated despite the absence of symptoms.
c. Surgery is indicated if the findings on myocardial perfusion imaging with sestamibi are normal.
d. Surgery is contraindicated based on the presence of left ventricular systolic dysfunction.
e. Surgery is indicated if the left ventricle remains dilated after 8 weeks of afterload-reducing
therapy with an ACE inhibitor. 

95. A 63-year-old woman presents to the clinic after an episode of sudden transient left-sided vision
loss. Symptoms lasted about 15 minutes with spontaneous resolution. Her past medical history is
significant for an anterior MI 3 weeks ago treated with PCI with bare metal stent (BMS) to the left
anterior descending artery (LAD). Other medical conditions include hypertension, type 2 diabetes
mellitus, and hyperlipidemia. Current medications are aspirin, prasugrel, atorvastatin, metformin, and
metoprolol. Physical examination is normal. There are no carotid bruits. Ophthalmologic examination is
within normal limits. An ECG reveals sinus rhythm with persistent ST elevations in V2 to V4.
What is the next step in management?
a. MRI of the brain with contrast
b. Carotid ultrasound
c. Left heart catheterization
d. Transthoracic echocardiogram
e. Exercise stress test with nuclear imaging 

96. A 64-year-old woman is evaluated for acute dyspnea 3 days after discharge following an inferior
myocardial infarction. When she was hospitalized, urgent coronary angiography showed single-vessel
coronary artery disease with occlusion of her mid-right coronary artery. She underwent successful
stenting of her right coronary artery, and was discharged on her third hospital day. Her ejection fraction
was 50% with inferior wall hypokinesis before discharge.
The patients dyspnea began 30 minutes ago. On physical examination, her pulse rate is 110/min,
respiration rate is 34/min, and blood pressure is 100/60 mm Hg. Jugular venous pressure is elevated at
10 cm H2O, crackles are heard halfway up both lung fields, a parasternal lift is appreciated, and there is a
new grade 3/6 systolic murmur at the left sternal border with an S3 gallop. The electrocardiogram shows
sinus tachycardia with Q waves and T wave inversions in leads II, III, and aVF, and is unchanged from the
discharge electrocardiogram.
A pulmonary artery catheter is placed, which shows the following:
Pressure (mm Hg) Oxygen Saturation (%)
Right atrium 12 (normal 2-7) 49 (normal 60-75)
Right ventricle 60/12 (normal 20-30/2-7) 78 (normal 60-75)
Pulmonary artery 60/32 (normal 20-30/10-15) 80 (normal 60-75)
Pulmonary capillary wedge 24 (normal <14) 98 (normal >93)
Which of the following is the most likely diagnosis?
a. Papillary muscle rupture
b. Recurrent myocardial infarction
c. Pericardial tamponade
d. Ventricular septal defect
e. Atrial septal defect

97. A 65 years old man admitted to hospital with sudden worsening shortness of breath preceding by
sharp chest pain and not relieved by resting. Before having chest pain, patient was walking about 400
meters. 3 days before patient having typical anginal chest pain and diaphoresis, but patient refuse to
seek medical assistant. Patient was an active smoker, had history of diabetes and uncontroled
hipertensbn. On physical examination, the patients found to have a blood pressure of 100/70 mmHg,
pulse 96 times per minutes. His JVP were raise and found s 3/6 harsh systolic murmur at the left sternal
border. Blood studies showed Hb 14 g/dl, Ht 43,7% , WBC 11900/uL, platelet 213.000/uL, ck 240, ck-mb
55.
Which of the following is the most likely anatomic diagnosis for this patient?
a. Acute myocardial infarction
b. Acute Long Edema
c. Ventricular Septal Rupture
d. Acute ischemic mitral regurgitation
e. Ventricular Septal Defect

98. A 65 years old man admitted to hospital with sudden worsening shortness of breath preceding by
sharp chest pain and not relieved by resting. Before having chest pain, patient was walking about 400
meters. 3 days before patient having typical anginal chest pain and diaphoresis, but patient refuse to
seek medical assistant. Patient was an active smoker, had history of diabetes and uncontrolled
hypertenstion. On physical examination, the patients found to have a blood pressure of 100/70 mmHg,
pulse 96 times per minutes. His JVP were raise and found s 3/6 harsh systole murmur at the left sternal
border. Blood studies showed Hb 14 g/dl, Ht 43,7% , WBC 11900/uL, platelet 213.000/uL, ck 240, ck-mb
55.
Which of the following statement is CORRECT in this case?
a. Primary PCI is the best choice for this patient
b. Hypothermic cardiopulmonary bypass with optimal myocardial protection must be done
c. Inotropic or vasopressor must given when blood pressure is low
d. Beta blocker is contraindicated
e. IABP has no benefit

99. A 65-year-old man presents after an arrest while eating at a local restaurant. On arrival, paramedics
documented ventricular fibrillation (VF), and he was successfully resuscitated. He has a history of
myocardial infarction (MI) and congestive heart failure (CHF). Serum electrolytes are remarkable only for
mild hypokalemia. MI is ruled out by ECG and serial blood tests of myocardial enzymes. Subsequent
evaluation includes cardiac catheterization, which shows severe three-vessel coronary artery disease
(CAD) and severe left ventricular (LV) systolic dysfunction. A nuclear myocardial perfusion scan shows a
large area of myocardial scar without significant viability in the territory of the left anterior descending
coronary artery. The decision is made to treat the CAD medically.
Which of the following is the best management strategy for his arrhythmia?
a. ICD implantion
b. Implantable cardioverter defibrillator (ICD) implantation if an electrophysiology (EP) study
shows inducible VT or VF
c. PO amiodarone
d. ?-Blocker medication

100. A 65-year-old man with a longstanding ischemic cardiomyopathy and LVEF 20% by an
echocardiogram obtained 1 month ago, was admitted to the hospital yesterday for increased shortness
of breath over the last 3 weeks. You are evaluating him in consultation with his internist. Overnight he
had made 1 liter of urine in response to diuretic therapy.
On examination, you find him to be in no acute distress at rest, blood pressure 110/70 mm Hg, pulse 96
bpm and regular, and his BMI is 38 kg/m2. He reports persistent shortness of breath when ambulating in
the corridors or in the supine position, the latter requiring two pillows to allow him to breathe
comfortably. The remainder of his examination is notable for JVP estimated to be 14 cm, S12 with no S3
or murmur, a benign abdomen with no hepatomegaly, and extremities which are entirely free of edema
and are warm to touch. His mental status is normal. Laboratory studies today include: sodium 140,
blood urea nitrogen 55, creatinine 1.9 (stable from admission), and hematocrit 34. Admission BNP was
75. A chest X-ray shows cardiomegaly with no evidence of pulmonary edema, pleural effusions, or
infiltrates.
At this point, which of the following is the most appropriate therapy to recommend?
a. order an echocardiogram and base further therapeutic decisions about the need for
diuretics or saline administration on the E/e' ratio.
b. Administration of IV Lasix 80 mg bid plus an infusion of dobutamine 2.5 mcg/kg/min to
provide inotropic support for treatment of acute on chronic systolic heart failure complicated
by renal insufficiency.
c. Administration of IV Lasix 80 mg bid for presumed acute on chronic systolic heart failure.
d. Administration of normal saline at 75 cc/hour for possible volume depletion.
e. Schedule for a right heart catheterization and adjust therapy based on invasively measured
hemodynamics.

101. A 66-year-old Caucasian woman is evaluated in the heart failure clinic for consideration of
advanced therapeutics, including possible heart transplantation. She has a longstanding history of
ischemic cardiomyopathy with an EF of 15% despite optimal medical therapy with beta-blockers, ACE
inhibitors, aldosterone antagonists, digoxin, and diuretics. She has a 60 pack-year history of smoking,
hypertension, and diet-controlled diabetes, and had a defibrillator/biventricular pacemaker device
placed 2 years ago. Her primary cardiologist was concerned about her worsening functional status, as
she becomes dyspneic with minimal activity. She performed "poorly" on a 6MWT, primarily limited by
dyspnea. A cardiopulmonary exercise test reveals peak V02 of 16.1 ml/kg/min. Which of the following
statements is TRUE regarding this patient and her functional capacity?
a. Based on her peak V02, she should be considered for heart transplantation at this time.
b. NYHA classification is an objective assessment of functional status with little interobserver
variability.
c. The 6MWT does not effectively discriminate the etiology of a patient's functional limitations,
and it therefore has no prognostic value in patients with heart failure.
d. Peak V02 is affected by age and gender, both of which should be accounted for when
assessing functional capacity.
102. 68-year-old woman presents to your office for initial evaluation. She has had progressive dyspnea
over th past 2 years. She has long-standing hyper tension and reports tobacco use (50 pack-years). She
has been treated with bosentan for idiopathic PAH (iPAH). Currently, she is assessed as having World
Health Organization (WHO) functional clas III limitations.
Evaluation revels the following:
Echocardiogram: ejection fraction 66%, grade 3 diastolic abnormality, moderate left ventricular
hypertrophy, no significant valvular disease
Right heart catheterization: right atrial pressure 8 mm Hg, right ventricular pressure 45/20 mm Hg,
pulmonary artery pressure 50/24 mm Hg, mean pulmonary artery pressure 33 mm Hg, pulmonary
capillary wedge pressure 25 mm Hg, and cardiac output 5.5 L/min
Which of the following scenarios may occur upon acute vasodilator challenge (pulmonary hypertension
reactivity test) in this patient?
a. Acute renal failure
b. Acute life-threatening pulmonary edema in the setting of a markedly elevated pulmonary
capillary wedge pressure
c. The patient may not respond and can then be considered a candidate for chronic calcium
channel blocker therapy
d. Altered mental status

103. A 68-year-old woman presents to the emergency room with severe orthopnea for the past 12
hours. She has a history of hypertension, osteoarthritis, and borderline diabetes mellitus. Exam revealed
irregular rate and rhythm with heart rate 115 bpm, blood pressure 190/100 mm Hg, jugular pressure 14
cm, and body mass index 38 kg/m2. Chest auscultation showed bilateral rales, while precordial exam
revealed an irregular and rapid heart rate with distant heart sounds and no discernible gallop. The
patient is given intravenous furosemide, diltiazem, and nitroglycerin infusions, with prompt
improvement in symptoms. Forty-eight hours later, she is now on oral losartan, diltiazem, aspirin, and
furosemide. Her heart rate is now irregular at 70 bpm, blood pressure is 130/65 mm Hg, venous
pressure is normal, but she still complains of moderate dyspnea on exertion.
Which of the following would you recommend?
a. Transesophageal echocardiogram-guided cardioversion.
b. Recommend bariatric surgery.
c. Nocturnal supplemental oxygen.
d. Switch diltiazem to atenolol.
e. Switch losartan to lisinopril. 

104. A 70 year old farmer presents with 3 days of intermittent chest pressure and dyspnea with minimal
exertion. He had one episode of nocturnal dyspnea 3 days prior. He is currently asymptomatic. His past
medical history includes HTN and hyperlipidemia. He is currently medicated with metoprolol 25 mg
twice daily and aspirin 325 mg daily.
On physical examination his BP is 140/85 mmHg and his HR is 76 bpm and regular. His JVP is normal. His
carotid upstrokes are normal and without bruits. His lungs are clear to auscultation. His heart has a
regular rate and rhythm. The apical impulse is in the normal location and of normal quality. The first and
second heart sounds are normal. There are no murmurs or gallops appreciated. The abdomen is soft
with no masses or bruits. The extremities have no clubbing, cyanosis, or edema, and the peripheral
pulses are normal.
The ECG shows nonspecific ST-T wave changes without frank elevation or depression. The chest X-ray is
interpreted as normal. CBC, electrolytes, and cardiac biomarkers are all negative.
The next best step is:
a. Either A or B (berdasarkan sumber jwbnnya ini tp dibolak balik optionnya)
b. Increase beta blockade and add nitrates, followed by noninvasive stress testing
c. Start therapy with tirofiban 0.1 mcg/kg/min
d. Pharmacologic stress testing
e. Diagnostic coronary angiography with possible percutaneous revascularization

105. A 70-year-old man comes to your office for evaluation before transurethral resection of the
prostate gland. He has a history of hypertension, feels well, leads an active life, and has no symptoms.
On physical examination, blood pressure is 140/70 mm Hg, and heart rate is 70/min and regular. Jugular
venous pressure and carotid upstroke are normal, and an S4 is heard. The patient has no S3, no
murmurs, and no peripheral edema. A preoperative chest radiograph shows cardiomegaly.
Echocardiogram performed to evaluate cardiomegaly shows left ventricular hypertrophy, with global left
ventricular dysfunction and an ejection fraction of 33%. No valvular abnormalities are noted. ACE
inhibitor therapy is initiated.
Which of the following medications should also be given to this patient?
a. Angiotensin receptor blocker
b. Digoxin
c. Diuretic
d. Spironolactone
e. ?-blocker

106. A 70-year-old man who has cough, shortness of breath, chronic cardiomyopathy, and an ejection
fraction of 30% is hospitalized with an exacerbation of pulmonary edema. He takes digoxin, an ACE
inhibitor, and a diuretic. On physical examination, blood pressure is 110/70 mm Hg, and heart rate is
70/min and regular. He has bibasilar crackles, an S3, and 1 + edema of the leg.
Which of the following is the most appropriate next step in the management of this patient?
a. Discontinue the ACE inhibitor and start treatment with an angiotensin receptor blocker.
b. Increase the dosage of digoxin.
c. Start long-term treatment with dobutamine.
d. Start treatment with a ?-blocker.
e. Increase diuresis

107. An 70 -year old woman is admitted via ambulance to emergency room. She is very dyspneic and
unable to give history. Medications found with her are as follows: furosemide 80 mg po od, atenolol 50
mg po od, warfarin, digoxin 0.125 mg po od. Routine observations are as follows: temperature 37 oC,
pulse 130 bpm, irregularly irregular, blood pressure 190/100 mmHg, respiratory rate 40 breath/min, 02
saturations are 88% on 10 IVmin 02 through rebreathe mask. Examination reveals central cyanosis and
cool peripheries. Auscultation of the chest reveals widespread inspiratory crepitations; pulses are absent
below the femoral arteries in both legs. The following investigations are available: arterial blood gases,
pH 7.12; p02 5.8 kPa; pC02 3,2 kPa; bicarbonate 6.0 mmol/L. Routine electrolytes; sodium 130 mmol/L;
potassium 5.5 mmol/L; creatinine 3.3 mg/L; glucose 6.0 mmol/L. Her CXR shows cardiomegaly, air
shadowing and Kerley B-lines. EKG recording demonstrate widespread ST depression and ST elevation in
lead aVR.
If you want to send this patient surgeon for CABG, there are some factors contribute to the
perioperative risk for operation. Angiogram is performed.
After discussed with the cardiac surgeon, he reluctant to perform operation. The reason not to do it
because of
a. Impaired LV function
b. Female gender
c. Renal disease
d. Recent acute coronary syndrome
e. Recent NSTEMI with critical myocardial ischemia

108. a 70 year old woman presents to your office with dyspnea and peripheral edema. On examination,
her BP is 180/70 mmHg and her pulse is 100 bpm. She has elevated jugular venous pressure, peripheral
edema of the ankles, and a fourth heart sound. All of the following would be reasonable to obtain in the
future, EXCEPT:
a. Serum electrolytes, complete blood count, and a urinalysis
b. Six minute walking test
c. ECG and chest x-ray
d. 24-hour Holter monitor
e. Echocardiogram

109. A 70-year-old woman who is new to your practice is evaluated because of progressive exertional
dyspnea for 3 months. She reports recent onset of orthopnea and paroxysmal nocturnal dyspnea. On
physical examination, blood pressure is 150/80 mm Hg, and heart rate is 95/min. She has normal sinus
rhythm, an S3 and an S4, no murmur, fine bibasilar crackles, and 1 + edema. Which of the following is
the most important test in the evaluation of this patient to guide initial therapy?
a. Measurement of the plasma B-type natriuretic peptide
b. Echocardiography
c. Chest radiography
d. Electrocardiography
e. Coronary angiography

110. A 72-year-old presents with sudden-onset chest pain at a local emergency department. He has a
past medical history significant for hypertension, hyperlipidemia, and gastroesophageal reflux disease.
On examination his BP is 95/60 mmHg and heart rate is 90 beats per minute and he is breathing at 90%
on ambient air. He reports this is the first time he has had any episode of chest pain. His
electrocardiogram (ECG) reveals ST elevation in VI to V4. The nearest hospital with percutaneous
coronary intervention (PCI) capability is 3 hours away.
What is the next step in management?
a. Administer unfractionated heparin, aspirin, and clopidogrel followed by transfer to PCI-
capable hospital.
b. Computed tomography (CT) of the chest with intravenous contrast.
c. Administer unfractionated heparin, aspirin, and clopidogrel and admit to hospital.
d. Perform fibrinolysis; low-molecular-weight heparin (LMWH), aspirin, and clopidogrel; and
transfer to the hospital for possible PCI.
e. Perform fibrinolysis; administer unfractionated heparin, aspirin, and clopidogrel; and admit to
hospital.

111. a 72-year-old woman is admitted to the hospital with acute STEMI. She has no other past medical
history. She underwent BMS to the left circumflex artery with good subsequent flow. She is now free of
chest pain and feels well. Echocardiogram revealed an estimated ejection fraction of 30%. In addition to
aspirin and clopidogrel.
what other medications must be considered in the patient's discharge medication regimen?
a. Atorvastatin, lisinopril, and carvedilol
b. Lisinopril, metoprobl, eplerenone, and niacin
c. Atorvastatin, lisinopril, carvedilol, and adolsterone antagonist
d. Atorvastatin, lisinopril, carvedilol, and warfarin

112. A 73-year-old woman comes to the emergency department because of a 2-hour history of severe
precordial chest pain and dyspnea. She has no significant past medical history, and is taking no
medication. An electrocardiogram shows 2-mm ST-segment elevation in leads V3-6. The patient is
promptly treated with chewable aspirin, intravenous heparin, and fibrinolysis with front- loaded tissue
plasminogen activator, following which her symptoms resolve and the ST segments return to normal.
She is admitted to the coronary care unit.
On physical examination, her pulse rate is 78/min and regular, and blood pressure is 120/76 mm Hg.
There are bibasilar crackles, and an S4 is present at the apex. She is prescribed aspirin, 325 mg daily,
atenolol, 50 mg daily, isosorbide mononitrate, 30 mg daily, furosemide, 40 mg orally, and intravenous
heparin with a goal of achieving an activated partial thromboplastin time of 55 to 80 seconds.
Laboratory studies 12 hours after admission show a markedly elevated serum troponin I level (50
ng/mL). A repeat electrocardiogram shows evidence of an evolving anterolateral myocardial infarction
with Q waves and ST-segment depression in leads V3-6. Telemetry over the first 12 hours shows four
episodes of asymptomatic, monomorphic nonsustained ventricular tachycardia, each episode lasting 6
to 15 beats. A transthoracic echocardiogram shows a large anterior wall motion abnormality and an
ejection fraction of 35%.
Which of the following should you also prescribe?
a. Amlodipine
b. Amiodarone
c. Diltiazem
d. Sotalol
e. Captopril

113. A 74-year-old man is resuscitated after an out-of-hospital cardiac arrest. He does not have a
myocardial infarction, but had two earlier myocardial infarctions. His left ventricular ejection fraction is
32%. An adenosine thallium scan shows anterior and inferior scarring, but no ischemia. Telemetry shows
that he has three to four episodes of nonsustained ventricular tachycardia daily. He has moderately
severe postanoxic encephalopathy. On medical therapy, he has no signs of heart failure. His laboratory
findings are normal. His family would like reasonable steps to be taken, but do not wish him to undergo
surgery, including placement of an implantable cardioverter defibrillator.
Which of the following drugs would be most effective in this patient?
a. Procainamide
b. Carvedilol
c. Diltiazem
d. Sotalol
e. Amiodarone

114. A 74-year-old woman is admitted with 3 hours of crushing substernal chest pain. She has a history
of left carotid occlusion with hemiparesis occurring 3 months ago. She also has a history of mild
hypertension, hyperlipidemia, and diabetes mellitus complicated by neuropathy and retinopathy. Her
medications include warfarin, 5 mg/d; atenolol, 25 mg/d; and pravastatin 20 mg/d. In the emergency
department, she has a ventricular fibrillation arrest. She is successfully cardioverted to sinus rhythm
after receiving 2 minutes of cardiopulmonary resuscitation. Her electrocardiogram shows sinus rhythm
with 3-mm ST-segment elevation in V2-6.The results of initial laboratory tests are within normal limits,
except for an elevated prothrombin time with an INR of 1.8.
Which of the following represent an absolute contraindication to the use of a thrombolytic agent in this
patient?
a. Cardiopulmonary resuscitation for 2 minutes
b. Patient on warfarin, with an INR of 1.8
c. Diabetic retinopathy
d. Left carotid occlusion with hemiparesis 3 months ago
e. Patient age >70 years

115. A 75-year-old woman Is referred urgently to the cardiology clinic. She had a myocardial infarction 4
years earlier, percutaneous coronary intervention with a stent for angina 12 months earlier and has had
two blackouts in the last month, 3 weeks apart. She tells you that in one occasion she was gardening and
trying to lift a heavy plant pot. She had no warning and suddenly found herself on the ground. She was
alert on recovery. There was no seizure-like activity. She does have exertional breathlessness although
she can manage 400m on the flat and a single flight of stairs. She has not had angina since her coronary
stent 12 months earlier. Occasionally she feels light-headed if she stand up too quickly. She is currently
taking aspirin, a beta-blocker, an ACE inhibitor, a loop diuretics and a statin. Her physical examination
reveals blood pressure 130/55 mmHg, resting pulse 55 bpm, regular, normal volume. The JVP is raised
by 2 cm, her apex beat is displaced to the lateral clavicular line, sixth intercostal space and there is a
systolic murmur heard all over the precordium and in the carotids. The lung field are clear and there is
mild pitting edema at the level of her shins.
The patient refuses immediate hospital admission but agrees to have a 24-hour Hotter monitor attached
and is scheduled for elective day-case electrophysiological study the following week. Coronary
angiography demonstrates a chronically occluded LAD artery, and a patent in her RCA. Programmed
ventricular stimulation is then performed, inducing monomorphic ventricular tachycardia at 200 bpm
with loss of consciousness and is promptly cardioverted with a single external 150 J biphasic shock. In
this case what is the most appropriate treatment?
a. ICD implantation
b. Correct the electrolyte imbalance
c. Stent implantation in occluded LAD artery
d. Amiodarone for preventing SCD
e. Send the patient to cardiac surgeon

116. A 75-year-old woman is referred urgently to the cardiology clinic. She had a myocardial infarction 4
years earlier, percutaneous coronary intervention with a stent for angina 12 months earlier and has had
two blackouts in the last month, 3 weeks apart. She tells you that in one occasion she was gardening and
trying to lift a heavy plant pot. She had no warning and suddenly found herself on the ground. She was
alert on recovery. There was no seizure-like activity. She does have exertional breathlessness although
she can manage 400m on the flat and a single flight of stairs. She has not had angina since her coronary
stent 12 months earlier. Occasionally she feels light-headed if she stand up too guickly. She is currently
taking aspirin, a beta-blocker, an ACE inhibitor, a loop diuretics and a statin. Her physical examination
reveals blood pressure 130/55 mmHg, resting pulse 55 bpm, regular, normal volume. The JVP is raised
by 2 cm, her apex beat is displaced to the lateral clavicular line, sixth intercostal space and there is a
systolic murmur heard all over the precordium and in the carotids. The lung field are clear and there is
mild pitting edema at the level of her shins.
All of the following would be the cause of blackout in this patient, EXCEPT?
a. A bradycardia caused by Stokes-Adam attack
b. Transient ischemic attack
c. Orthostatic hypotension
d. A tachycardia caused by ventricular tachycardia
e. Severe aortic stenosis

117. A 75-year-old woman is referred urgently to the cardiology clinic. She had a myocardial infarction 4
years earlier, percutaneous coronary intervention with a stent for angina 12 months earlier and has had
two blackouts in the last month, 3 weeks apart. She tells you that in one occasion she was gardening and
trying to lift a heavy plant pot. She had no warning and suddenly found herself on the ground. She was
alert on recovery. There was no seizure-like activity. She does have exertional breathlessness although
she can manage 400m on the flat and a single flight of stairs. She has not had angina since her coronary
stent 12 months earlier. Occasionally she feels light-headed if she stand up too guickly. She is currently
taking aspirin, a beta-blocker, an ACE inhibitor, a loop diuretics and a statin. Her physical examination
reveals blood pressure 130/55 mmHg, resting pulse 55 bpm, regular, normal volume. The JVP is raised
by 2 cm, her apex beat is displaced to the lateral clavicular line, sixth intercostal space and there is a
systolic murmur heard all over the precordium and in the carotids. The lung field are clear and there is
mild pitting edema at the level of her shins.
If you choose ICD implantation as a class I indication in patient like this, the reason you choose this one
are based on all of the following EXCEPT?
a. Sustain monomorphic ventricular tachycardia
b. After has had a non-hemorrhagic stroke with atrial fibrillation
c. Ejection fraction <35%
d. Ischemic heart disease for arrhythmia prevention
e. Syncope of unknown origin

118. A 75-year-old woman is referred urgently to the cardiology clinic. She had a myocardial infarction 4
years earlier, percutaneous coronary intervention with a stent for angina 12 months earlier and has had
two blackouts in the last month, 3 weeks apart. She tells you that in one occasion she was gardening and
trying to lift a heavy plant pot. She had no warning and suddenly found herself on the ground. She was
alert on recovery. There was no seizure-like activity. She does have exertional breathlessness although
she can manage 400m on the flat and a single flight of stairs. She has not had angina since her coronary
stent 12 months earlier. Occasionally she feels light-headed if she stand up too guickly. She is currently
taking aspirin, a beta-blocker, an ACE inhibitor, a loop diuretics and a statin. Her physical examination
reveals blood pressure 130/55 mmHg, resting pulse 55 bpm, regular, normal volume. The JVP is raised
by 2 cm, her apex beat is displaced to the lateral clavicular line, sixth intercostal space and there is a
systolic murmur heard all over the precordium and in the carotids. The lung field are clear and there is
mild pitting edema at the level of her shins.
What is the most appropriate immediate investigation should be performed?
a. Lying and standing blood pressure
b. 12- lead ECG
c. Chest X-ray
d. Echocargogram
e. Carotid sinus massage
119. A 78-year-old diabetic male is referred to your clinic for preoperative evaluation prior to left knee
replacement. He currently can only walk 1 to 2 blocks before stopping, but he is limited by knee pain
and denies angina or shortness of breath. He has an adenosine sestamibi stress test that demonstrates a
small area of ischemia at the apex. He did not note any discomfort during the test. He is currently on 81
mg aspirin daily and has adequate beta blockade with metoprolol. His vitals are:
HR: 62 bpm?BP: 118/70 mmHg?Lipids: LDL 68 mg/dL, HDL 45 mg/dL, TG 98 mg/dL
Your next step is:
a. Increase aspirin to 325 mg daily and continue beta blocker
b. Proceed to coronary angiogram to perform PCI on the LAD
c. Tell patient that nothing further evaluation is needed at this time
d. Advise patient to postpone surgery for further diagnostic testing
e. Proceed to coronary angiogram to define the anatomy
120. A 78-year-old man calls 911 because he has sudden-onset chest pain. On EMS arrival he has a BP of
120/80 mmHg and a heart rate of 80 beats per minute. His lungs are clear to auscultation and he has a
left ventricular (LV) S4 noted. ECG done on the field confirms an anterior MI with ST elevation from VI to
V4. He is given an aspirin and a sublingual nitroglycerin and transfer is initiated. Within 5 minutes while
en route his BP is noted to be 60 mmHg and a saline bolus is initiated. Cardiovascular examination is
unchanged and no murmurs are noted.
Which of the following scenarios may likely explain his observed hemodynamic deterioration?
a. Anaphylaxis reaction from aspirin
b. Massive pulmonary embolism
c. Right ventricular (RV) infarction
d. Recent exposure to a phosphodiesterase 5 inhibitor
e. Hypotension resulting from bleeding due to administration of aspirin

121. A 79-year-old man with diabetes, HTN, chronic renal insufficiency, and ischemic cardiomyopathy
was recently admitted with CHF exacerbation. At home, he takes captopril, 75 mg t.i.d.; digoxin, 0.125
mg per day; furosemide, 60 mg b.i.d.; aspirin; and atorvastatin calcium (Lipitor). When admitted, he was
in heart failure with elevated neck veins and S3. During his admission, he was diuresed with IV
furosemide and metolazone. His baseline creatinine was 1.7 and now is 2.5, with blood urea nitrogen of
100.
What is your next step?
a. Stop captopril.
b. Stop aspirin and ACE inhibitor
c. Stop diuretics.
d. Rule out renal artery stenosis. 

122. An 80 year old hypertensive male referred to cardiology clinic with stable NYHA FC III angina for 3
months treated with aspirin, metoprolol succinate 150 mg daily, isosorbide mononitrate 120 mg daily,
and simvastatin 40 mg daily. On exam, the heart rate is 57, the blood pressure is 98/60 mmHg, and the
cardiopulmonary exam is unremarkable. Resting ECG is within normal limits. An exercise stress test is
significant for 2 mm horizontal ST depression and exercise limiting chest discomfort at 6 METs.
The Duke Treadmill Score for this patient is:
a. -14, intermediate risk
b. -9, intermediate risk
c. -9, high risk
d. -14, high risk
e. -10, intermediate risk 
123. An 80 year old hypertensive male referred to cardiology clinic with stable NYHA FC III angina for 3
months treated with aspirin, metoprolol succinate 150 mg daily, isosorbide mononitrate 120 mg daily,
and simvastatin 40 mg daily. On exam, the heart rate is 57, the blood pressure is 98/60 mmHg, and the
cardiopulmonary exam is unremarkable. Resting ECG is within normal limits. An exercise stress test is
significant for 2 mm horizontal ST depression and exercise limiting chest discomfort at 6 METs.
The most appropriate next step would be:
a. Echocardiography
b. Cardiac catheterization
c. Add calcium channel blocker
d. Increase nitrate dose
e. Increase beta blocker dose

124. an 80-year-old man with chronic AFib of 15 years’ duration is admitted with recurrent episodes of
dizziness and a recent episode of syncope. He has normal LV function and no evidence of CAD. In-
hospital telemetry confirms the presence of slow ventricular rate and frequent pauses (4 seconds) that
correlate with his lightheadedness. His medications consist of warfarin (Coumadin).
A decision was made to proceed with a permanent pacemaker.
Which would be the most suitable pacing modality?
a. Dual-chamber system programmed to DDDR
b. Single-chamber system in the ventricle programmed to WIR
c. Single-chamber system in the atrium programmed to AAIR
d. Dual-chamber system programmed to DDIR
e. Dual-chamber system programmed to DDDR with mode switching

125. a patient presents with regular narrow QRS tachycardia. A 12-lead electrocaediogram (ECG)
demonstrate an r? in lead V1 that was not seen on ECG when the patient was in sinus rhythm. An
esophageal electrode shows a 1:1 atrial-to-ventricular relationship during tachycardia. The ventricular
(VA) interval is measured as 55 milliseconds.
Which of the following is the most likely diagnosis?
a. Permanent junctional reciprocating tachycardia (PJRT)
b. Orthodromic atrioventricular reentrant tachycardia (AVRT)
c. AV nodal reentrant tachycardia (AVNRT)
d. Atrial tachycardia

126. A patient with NYHA class III HF was hospitalized 2 months ago for an exacerbation of his HF. The
patient wwas discharged on lisinopril , furosemide, and digoxin. His lungs are clear and his vital signs are
as follows: BP, 105/56 mmHg; heart rate, 84 bpm; and respiration rate, 18.
Which regimen is most appropriate to initiate in this patient?
a. Metoprolol tartrate 50 mg twice daily
b. Carvedilol 3.125 mg twice daily
c. Bisoprolol 2.5 mg daily
d. Carvedilol 25 mg twice daily

127. A previously healthy 36-year-old African-American man is admitted from the emergency
department with newly diagnosed congestive heart failure. An echocardiogram is obtained on admission
and reveals significant four-chamber dilatation, an EF of 25%, and an LV end-diastolic dimension of 7.3
cm. A left heart catheterization reveals no evidence of obstructive coronary disease. The patient does
not use alcohol, cocaine, or other illicit drugs, and he has no recent history of viral illness. While
discussing his medical history, he notes that his father died of heart failure at age 38. You consider the
possibility that the patient has some form of familial cardiomyopathy.
Which of the following statements is TRUE concerning familial cardiomyopathies?
a. A thorough family history should be taken, including at least three generations.
b. It is estimated that <5% of idiopathic dilated cardiomyopathies are familial in origin.
c. Most familial cardiomyopathies exhibit autosomal recessive inheritance patterns.
d. To identify the accurate diagnosis, all family members should be tested first for multiple
mutations associated with the disease.

128. According to the ACCF/AHA 2009 focused update of the 2005 guidelines for the diagnosis and
management of heart failure in adults, which of the following is a Class 1 recommendation for the
prevention of chronic HF in patients at high risk of developing HF (Stage A)?
a. Angiotensin-converting enzyme inhibitors can be useful to prevent HF in patients at high risk
for developing HF due to a history of atherosclerotic vascular disease, diabetes mellitus, or
hypertension with associated cardiovascular risk factors
b. Routine use of nutritional supplements to prevent the development of structural heart
disease is recommended.
c. Angiotensin n receptor blockers can be useful to prevent HF in patients at high risk for
developing HF who have a history of atherosclerotic vascular disease, diabetes mellitus, or
hypertension with associated cardiovascular risk factors.
d. Thyroid disorders should be treated in accordance with contemporary guidelines.
e. Beta-blockers can be useful to prevent HF in patients at high risk for developing HF who have
a history of atherosclerotic vascular disease, diabetes mellitus, or hypertension with
associated cardiovascular risk factors.

129. After appropriate diagnostic workup and medical therapy are commenced, this patient is found to
have a 95% stenosis in the middle LAD coronary artery. This was successfully treated with an
intracoronary bare metal stent.
With regard to this patient:
a. There is a 4% to 6% risk of in-stent restenosis over the next 6 months
b. Aspirin 81 mg plus warfarin adjusted to an INR of 2.0 to 2.5 should be commenced
c. Noninvasive stress testing is required at 3 to 6 months following the percutaneous procedure
regardless of the patient's symptom status
d. Aspirin 325 mg and clopidogrel 75 mg daily should be commenced

130. Among patients with class n angina and one- or two-vessel disease, PCI is indicated for which of the
following:
a. Prevention of MI
b. To alleviate asymptomatic ischemia
c. To prevent progression of CAD o
d. To improve symptoms
e. Prevention of death

131. An 70 -year old woman is admitted via ambulance to emergency room. She is very dyspneic and
unable to give history. Medications found with her are as follows: furosemide 80 mg po od, atenolol 50
mg po od, warfarin, digoxin 0.125 mg po od. Routine observations are as follows: temperature 37oC,
pulse 130 bpm, irregularly irregular, blood pressure 190/100 mmHg, respiratory rate 40 breath/min, 02
saturations are 88% on 10 U/min 02 through rebreathe mask. Examination reveals central cyanosis and
cool peripheries. Auscultation of the chest reveals widespread inspiratory crepitations; pulses are absent
below the femoral arteries in both legs. The following investigations are available: arterial blood gases,
pH 7.12; p02 5.8 kPa; pC02 3,2 kPa; bicarbonate 6.0 mmol/L. Routine electrolytes; sodium 130 mmol/L;
potassium 5.5 mmol/L; creatinine 3.3 mg/dl; glucose 108 mg/dl. Her CXR shows cardiomegaly, air
shadowing and Kerley B-lines. EKG recording demonstrate widespread ST depression and ST elevation in
lead aVR.
All of the following are the findings on the arterial blood gases, EXCEPT?
a. A ventilation perfusion mismatch
b, Metabolic acidocis relelated to renal failure
c. Type 1 respiratory failure
d. Profound lactic acidosis
e. Severe hyperkalemia due to renal dysfunction

132. An 70 -year old woman is admitted via ambulance to emergency room. She is very dyspneic and
unable to give history. Medications found with her are as follows: furosemide 80 mg po od, atenolol 50
mg po od, warfarin, digoxin 0.125 mg po od. Routine observations are as follows: temperature 37oC,
pulse 130 bpm, irregularly irregular, blood pressure 190/100 mmHg, respiratory rate 40 breath/min, 02
saturations are 88% on 10 L/min 02 through rebreathe mask. Examination reveals central cyanosis and
cool peripheries. Auscultation of the chest reveals widespread inspiratory crepitations; pulses are absent
below the femoral arteries in both legs. The following investigations are available: arterial blood gases,
pH 7.12; p02 5.8 kPa; pC02 3,2 kPa; bicarbonate 6.0 mmol/L. Routine electrolytes; sodium 130 mmol/L;
potassium 5.5 mmol/L; creatinine 3.3 mg/dl; glucose 108 mg/dl. Her CXR shows cardiomegaly, air
shadowing and Kerley B-lines. EKG recording demonstrate widespread ST depression and ST elevation in
lead aVR.
Additional findings on cardiovascular examination are of a prominent apex beat, which is displaced to
the mid-axillary line. There is a soft systolic murmur present throughout the precordium and a gallop
rhythm.
What the most likely causes in this woman is after evaluate all of the history and clinical examination?
a. Critical myocardial ischemia ®
b. Flash pulmonary edema associated with renal failure
c. Acute on chronic renal failure
d. Dilated cardiomyopathy
e. Acute myocardial infarction

133. An 70 -year old woman Is admitted via ambulance to emergency room. She is very dyspneic and
unable to give history. Medications found with her are as follows : furosemide 80 mg po od, atenobl 50
mg po od, warfarin, digoxin 0.125 mg po od. Routine observations are as follows: temperature 37oC,
pulse 130 bpm, irregularly irregular, blood pressure 190/100 mmHg, respiratory rate 40 breath/min, 02
saturations are 88% on 10 L/min 02 through rebreathe mask. Examination reveals central cyanosis and
cool peripheries. Auscultation of the chest reveals widespread inspiratory crepitations; pulses are absent
below the femoral arteries in both legs. The following investigations are available: arterial blood gases,
pH 7.12; p02 5.8 kPa; pC02 3,2 kPa; bicarbonate 6.0 mmol/L. Routine electrolytes; sodium 130 mmol/L;
potassium 5.5 mmol/L; creatinine 3.3 mg/L; glucose 6.0 mmol/L. Her CXR shows cardiomegaly, air
shadowing and Kerley B-lines. EKG recording demonstrate widespread ST depression and ST elevation in
lead aVR.
If you want to send this patient surgeon for CABG, there are some factors contribute to the
perioperative risk for operation. Angiogram is performed.
After discussed with the cardiac surgeon, he reluctant to perform operation. The reason not to do it
because of
a. Recent NSTEMI with critical myocardial ischemia
b. Female gender
c. Recent acute coronary syndrome
d. Renal disease
e. Impaired LV function

134, An 70 -year old woman is admitted via ambulance to emergency room. She is very dyspneic and
unable to give history. Medications found with her are as follows: furosemide 80 mg po od, atenolol 50
mg po od, warfarin, digoxin 0.125 mg po od. Routine observations are as follows: temperature 37oC,
pulse 130 bpm, irregularly irregular, blood pressure 190/100 mmHg, respiratory rate 40 breath/min, 02
saturations are 88% on 10 L/min 02 through rebreathe mask. Examination reveals central cyanosis and
cool peripheries. Auscultation of the chest reveals widespread inspiratory crepitations; pulses are absent
below the femoral arteries in both legs. The following investigations are available: arterial blood gases,
pH 7.12; p02 5.8 kPa; pC02 3,2 kPa; bicarbonate 6.0 mmol/L. Routine electrolytes; sodium 130 mmol/L;
potassium 5.5 mmol/L; creatinine 300 umol/L; glucose 6.0 mmol/L. Her CXR shows cardiomegaly, air
shadowing and Kerley B-lines. EKG recording demonstrate widespread ST depression and ST elevation in
lead aVR.
This patient is given 40 mg of furosemide intravenously and commenced on intravenous nitrates. One
hour after comm encing therapy she remains acutely unwell. 02 saturation are 86% on 10 L 02 via a
rebreath mask, blood pressure is now 140/70 mmHg and the respiratory rate is 35 breaths/min; there
has been no urine output.
What other interventions would be the most valuable you consider?
a. Intra aortic balloon counterpulsation Hemofiltration
b. hemofiltration
c. Administration of cardiac glycoside
d. Continuous positive airways pressure
e. High-dose intravenous diuretics

1 35. An 70 -year old woman is admitted via ambulance to emergency room. She is very dyspneic and
unable to give history. Medications found with her are as follows : furosemide 80 mg po od, atenolol 50
mg po od, warfarin, digoxin 0.125 mg po od. Routine observations are as follows: temperature 37oC,
pulse 130 bpm, irregularly irregular, blood pressure 190/100 mmHg, respiratory rate 40 breath/min, 02
saturations are 88% on 10 IVmin 02 through rebreathe mask. Examination reveals central cyanosis and
cool peripheries. Auscultation of the chest reveals widespread inspiratory crepitations; pulses are absent
below the femoral arteries in both legs. The following investigations are available: arterial blood gases,
pH 7.12; p02 5.8 kPa; pC02 3,2 kPa; bicarbonate 6.0 mmol/L. Routine electrolytes; sodium 130 mmol/L;
potassium 5.5 mmol/L; creatinine 300 umol/L; glucose 6.0 mmol/L. Her CXR shows cardiomegaly, air
shadowing and Kerley B-lines. EKG recording demonstrate widespread ST depression and ST elevation in
lead aVR.
What is the probable working diagnosis?
a. Hypertensive heart failure
b. Pulmonary emboli and severe pneumonia
c. Bilateral pleural effusions
d. Pulmonary emphysema and cor-pulmonale
e. Pulmonary edema and acidosis

136. An 80 year old hypertensive male referred to cardiology clinic with stable NYHA FC III angina for 3
months treated with aspirin, metoprobl succinate 150 mg daily, isosorbide mononitrate 120 mg daily,
and simvastatin 40 mg daily. On exam, the heart rate is 57, the blood pressure is 98/60 mmHg, and the
cardiopulmonary exam is unremarkable. Resting ECG is within normal limits. An exercise stress test is
significant for 2 mm horizontal ST depression and exercise limiting chest discomfort at 6 METs.
The most appropriate next step would be:
a. Echocardiography
b. Increase nitrate dose
c. Cardiac catheterization
d. Increase beta blocker dose
e. Add calcium channel blocker

137. An 80-year-old man presents to your office with complaints of chest tightness when climbing up a
flight of stairs. His past medical history is unremarkable. On physical examination, he is in no acute
distress. BP is 140/80 mmHg; pulse is 78 bpm and regular. Chest is clear. Carotid upstrokes are
diminished. The PMI is sustained, but not displaced. A fourth heart sound is present. The second heart
sound is diminished and single. A bud late-peaking systolic murmur is heard, loudest at the second
intercostal space, radiating to the neck.
The above patient is found to have an aortic valve area of 0.7 cm 2 with a mean gradient of 60 mmHg.
Following catheterization, he develops massive upper gastrointestinal bleeding. Endoscopy reveals a
gastric ulcer with a bleeding vessel at its base. Cauterization is performed, which temporarily stops the
bleeding. However, the bleeding recurs and urgent partial gastrectomy is recommended. He complains
of chest pain during these bleeding episodes.
What is the best course of action?
a. Refer for percutaneous balloon valvubplasty, followed by gastrectomy.
b. Proceed to AVR first.
c. Start nitroprusside and proceed with gastric surgery.
d. Proceed with gastric surgery directly.

138. an 80-year-old man with chronic AFib of 15 years’ duration is admitted with recurrent episodes of
dizziness and a recent episode of syncope. He has normal LV function and no evidence of CAD. In-
hospital telemetry confirms the presence of slow ventricular rate and frequent pauses (4seconds) that
correlate with his lightheadedness. His medications consist of warfarin sodium (Coumadin).
The most appropriate course of action includes which of the following?
a. ICD implant
b. Reassuring the patient and instructing him to come back if he has recurrence of syncope
c. EP testing to evaluate for ventricular arrhythmia
d. Permanent pacemaker implant
e. Prolonged monitoring with a loop recorder

137. An 80-year-old man presents to your office with complaints of chest tightness when climbing up a
flight of stairs. His past medical history is unremarkable. On physical examination, he is in no acute
distress. BP is 140/80 mmHg; pulse is 78 bpm and regular. Chest is clear. Carotid upstrokes are
diminished. The PMI is sustained, but not displaced. A fourth heart sound is present. The second heart
sound is diminished and single. A loud late-peaking systolic murmur is heard, loudest at the second
intercostal space, radiating to the neck.
The above patient is found to have an aortic valve area of 0.7 cm2 with a mean gradient of 60 mmHg.
Following catheterization, he develops massive upper gastrointestinal bleeding. Endoscopy reveals a
gastric ulcer with a bleeding vessel at its base. Cauterization is performed, which temporarily stops the
bleeding. However, the bleeding recurs and urgent partial gastrectomy is recommended. He complains
of chest pain during these bleeding episodes.
What is the best course of action?
a. Proceed with gastric surgery directly.
b. Proceed to AVR first.
c. Start nitroprusside and proceed with gastric surgery.
d. Refer for percutaneous balloon valvuloplasty, followed by gastrectomy.

138. An 80-
year-old undergoes dual-chamber pacemaker placement for CHB. Excellent ventricular and atrial
capture thresholds were obtained at the time of the implant. The pacemaker programmed parameters
are as follows: mode, DDD; lower rate, 70 bpm; upper rate, 130 bpm; atrial sensitivity, 0.25 mV (most
sensitive setting); ventricular sensitivity, 1.00 mV; AV delay, 175 milliseconds; pace/sense configuration,
bipolar.
The next day, the following rhythm strip was recorded: This rhythm strip shows
a. ventricular noncapture.
b. ventricular undersensing.
c. atrial noncapture.
d. atrial underselling.
e. normal pacemaker function for the programmed parameters.

139. An 85 year old hypertensive male referred to cardiology clinic with stable NYHA FC III angina for 3
months treated with aspirin, bisoprolol 5 mg daily, isosorbide mononitrate 120 mg daily, and simvastatin
40 mg daily. On exam, the heart rate is 57, the blood pressure is 98/60 mmHg, and the cardiopulmonary
exam is unremarkable. Resting ECG is within normal limits. An exercise stress test is significant for 2 mm
horizontal ST depression and exercise limiting chest discomfort at 6 METs (4 minutes duration of
exercise test).
The most appropriate next step would be:
a. Increase beta blocker dose
b. Add calcium channel blocker
c. Cardiac catheterization
d. Increase nitrate dose
e. Echocardiography

140. an 85-year-old man presents with increasing intensity of his typical angina pain associated with
shortness of breath. His ECG is unchanged from prior tracings. He has a history of established coronary
artery disease and has previously refused revascularization. A decision is made to manage him
conservatively.
Which of the following interventions would be considered inappropriate?
a. Treatment with clopidogrel 300 mg followed by 75 mg daily
b. Initiation of anticoagulation with fondaparinux
c. Continue treatment with aspirin
d. Initiate treatment with prasugrel bolus 60 mg followed by 5 mg once daily
e. Increase the dosage of prior ?-blocker therapy

141. An active 40-year-old premenopausal woman is seen in the emergency department because of
substernal chest pressure. The pressure started during an argument with her husband, decreased as she
was transported to the hospital, and was gone by the time she was examined. She has not had this
symptom before. Her heart rate is 90/min and blood pressure is 150/90 mm Hg. Findings on physical
examination are otherwise normal. The findings on electrocardiogram and chest radiograph are normal.
Serum creatine kinase-MB and troponin I levels are normal at baseline and 6 hours later. Her only risk
factor for coronary artery disease is mild obesity. Her body mass index is 31.
Which of the following is the best course of action?
a. Admit the patient to the cardiac care unit for monitoring
b. Discharge the patient and perform an exercise nuclear perfusion study within 72 hours
c. Discharge the patient and perform an exercise electrocardiogram within 72 hours
d. Discharge the patient and perform an exercise echocardiogram within 72 hours
e. Discharge the patient and recommend follow-up with her primary care physician

142. B. T. is a 56-year-old woman with long-standing HTN that is difficult to control. She is currently
being treated with amlodipine 10 mg daily. Lisinopril 40 mg daily, hydrochlorothiazide 25 mg daily, and
clonidine 0.4 mg three times daily. She presented to the emergency room, and her initial BP was
200/110 mmHg. She states she had run put of one of her medications.
which one of her medications would most likely be implicated in causing hypertensive urgency?
a. Lisinopril
b. Clonidine
c. Hydrochlorothiazide
d. Amlodipine

143. Based on epidemiological studies, which of the following risk factors has the lowest relative risk
range for developing lower extremity peripheral arterial disease?
a. Hypercholesterolemia
b. Hypertension
c. Diabetes
d. Hyperhomocysteinemia
e. Smoking
144. B-Blockers may be even more widely used in the future because of their likely ability to prevent
which of these?
a. Control hypertension
b. Peptic ulcer disease
b. Myocardial infarction
d. Peripheral vascular disease
c. Hyperuricemia and gout

145. CASE 1.
A 45-year old business executive presents to A&E with a 2-hour history of central crushing chest pain
and breathlessness. He is a non-smoker, previously very fit and well and attends a gym four times a
week. There is no family history of ischemic heart disease. His cholesterol measured at an insurance
medical was 3,3 mmol/L. His observations on admission are as follows; pulse 105 bpm; blood pressure
80/50 mmHg; 02 saturations 90% on room air. He is apyrexial. An ECG is performed and shown sinus
tachycardia of 105 bpm. Right-axis deviation and non-specific T-wave inversion in leads III, aVF, V2-V4.
As a business executive,
All of the following aspects of the history and examination would help you to establish diagnosis,
EXCEPT ?
a. Accentuated pulmonary second sound
b. The patient may have recently been on a long-haul flight
c. Family history of thromboembolic disease
d. Radiographic evidence of pulmonary edema
e. Absence of exertional angina

146. Classification of myocardial infarction type 2 according to the universal definition of myocardial
infarction?
a. MI associated with coronary artery bypass graft surgery
b. MI associated with percutaneous coronary intervention
c. Spontaneous MI result from a primary coronary event (coronary plague errosion, rupture, or
dissection)
d. Sudden unexpected cardiac death
e. MI associated with ischemia secondary to either increased oxygen demand or decreased
supply

147. D. L. is a 76-year-old white man with a past medical history significant for DM type 2 and HTN.
Chronic AFib was recently diagnosed with coronary artery disease (CAD) and hypercholesterolemia and
he was initiated on gemfibrozil 600 mg twice daily and atorvastatin 40 mg daily. His other medications
include glyburide, metoprolol, furosemide, levothyroxine, insulin, and aspirin. Two weeks later, he
began to experience pain in his right calf, with pain and stiffness throughout his back, buttocks, and
thigh. After another week, he was admitted to the hospital with similar heightened symptoms. On
admission, his blood urea nitrogen was elevated, and the urinalysis showed orange, cloudy urine;
protein, greater than 300; glucose, greater than 1,000; ketones, 2+; hemoglobin, 3+; red blood cell
count, 6 to 10; and myoglobin, 1,367.
Which of the following statements is true?
a. If nicotinic acid, rather than gemfibrozil, had been used for hypercholesterolemia, this
reaction would have been prevented.
b. Forced diuresis with urine alkalinization and discontinuation of gemfibrozil and atorvastatin
are indicated for this patient.
c. Atorvastatin is contraindicated in a patient with DM type 2 and HTN.
d. The patient is experiencing rhabdomyolysis secondary to the drug interaction of
atorvastatin and glyburide. 

148. During the cardiac catheterization of a person who was found to be normal, the blood withdrawn
through the catheter had an oxygen saturation of 60%, and the recorded pressure oscillated each
heartbeat between 14 and 26 mmHg. Most likely, the catheter tip was located in the:
a. Pulmonary artery.
b. Right atrium.
c. Azygous vein.
d. Coronary sinus.
e. Foramen ovale.
149. each of the following statements regarding high-sensitivity C-Reactive protein (HsCRP) is true:
a. Statins reduce hsCRP in a manner directly related to their low density lipoprotein-lowering
effect
b. An elevated level of hsCRP is predictive of the onset of metabolic syndrome
c. An hsCRP level >2 mg/L in a patient with unstable angina is associated with an increased risk
of recurrent coronary events
d. The cardiovascular benefit of aspirin therapy appears to be greatest in patients with
elevated hsCRP levels
e. Included in Global Risk Score, a predictive risk score tools to predict cardiovascular mortality
in asymptomatic patients

150. evaluation revels the following:


Echocardiogram: ejection Fraction 66%, grade 3 diastolic abnormality, moderate left ventricular
hypertrophy, no significant valvular disease.
Right heart catheterization: right atrial pressure 8 mm Hg, right ventricular pressure 45/20 mm Hg,
pulmonary artery pressure 50/24 mm Hg, mean pulmonary artery pressure 33 mm Hg, pulmonary
capillary wedge pressure 25 mm Hg, and cardiac output 5.5 L/min
Based on your review of her evaluation findings,
which of the following represents the appropriate diagnosis?
a. Pulmonary Vein Hypertension
b. Pulmonary arterial hypertension
c. Chronic obstructive pulmonary disease (COPD)
d. Viral cardiomyopathy

151. factors that will reduced cardiac output?


a. Baroreceptor stimulation
b. Phosphodiesterase inhibitors (milrinone, theophylline)
c. Digitalis glycoside
d. Sympathetic tone
e. Volume load

152. False statement regarding the assessment microvascular obstruction by CMR:


a. A dark core of hypoenhancement within the infarct tissue is taken as a region of coronary
microvascular obstruction
b. Images obtained 10-15 min after injecting a gadolinium-based contrast
c. The index of microcirculatory
d. Infarcted tissue is seen as a region of hypo enhancement

153. HF prognosis correlates MOST STRONGLY with which of the following?


a. Resting EF
b. Genetic predisposition
c. NYHA class
d. Diastolic dysfunction
e. Impaired renal function

154. Hydration prior to contrast angiography is useful to:


a. Reduce the chance of pulmonary edema
b. Reduce the chance of contrast nephropathy
c. Reduce the chance of contrast nephropathy & Reduce the chance of thrombotic episodes in
cyanotics
d. Reduce the chance of thrombotic episodes in cyanotics

155. if a drug is given that acts specifically to increase myocardial contractility:


a. Arterial pulse pressure will decrease.
b. Mean arterial pressure will decrease.
c. Central venous presage will decrease.
d. Stroke volume will decrease.
e. Capillary blood flow will decrease. 

156. L. M. has been receiving digoxin 0.25 mg PC) tablets daily. Her serum drug level is 1.8 ng/ml_. She is
no longer able to take PO medications and needs to receive digoxin IV.
By what percentage do you need to decrease the dose to maintain the current digoxin level?
a. 25%
b. 50%
c. 40%
d. 10%

157. M. M. is a 39-year-old man with an inferior wall non-ST-segment-elevation MI. He has a history of
poorly controlled HTN and diabetes mellitus (DM). You initiate aspirin, clopidogrel, and atorvastatin. His
baseline serum creatinine is 3.4 mg/dL and you estimate his creatinine clearance to be 25 mL/min.
What dose of enoxaparin would you choose?
a. 1 mg/kg daily
b. Enoxaparin is not indicated at this time
c. Fondaparinux is safer to use in M. M.
d. 1 mg/kg every 12 hours

158. Mitral stenosis is considered severe when the:


a. Resting mean pressure gradient is >= 5 mmHg
b. No correct answer
c. PHT is >= 200 msec
d. Mitral valve area area is >= 1.5cm2
e. Deceleration time >= 660ms

159. Mr. Sujarwo. is an obese 54-year-old man (BMI = 35) with a history significant for obstructive sleep
apnea and multiple joint complaints. A lower-extremity arterial duplex study, performed to work up the
possibility of claudication, reveals noncritical peripheral arterial disease, with bilateral ankle-brachial
indices (ABIs) of 0.8. He denies ongoing exertional chest discomfort or dyspnea, but lives a fairly
sedentary lifestyle. Physical exam reveals an obese man with normal blood pressure (BP), an
unremarkable cardiac exam, and 1+ dorsalis pedis pulses. His fasting lipids are as follows: TC 240 mg/dl_,
TG 250 mg/dl_, HDL-C 35 mg/dL, LDL-C (calculated) 155.
Initial therapy should include:
a. TLC plus a statin, with the goal of reducing LDL-C to <100 mg/dL (<70 mg/dL optional)
b. TLC plus statin, as well as niacin or a fibrate, with the goals of reducing LDL-C < 100 mg/dL and
TG <150 mg/dL
c. TLC plus a statin, with the goal of reducing LDL-C to < 130 mg/dL
d. TLC (therapeutic lifestyle changes) only
e. TLC plus niacin or a fibrate, with the goals of reducing LDL-C < 100 mg/dL and TG < 150
mg/dL 

160. N.M. is a 75-year-old woman with a long-standing history of HF secondary to viral cardiomyopathy.
She presents to the outpatient clinic for routine follow-up. On examination, she was short of breath and
reported increasing orthopnea. She was admitted to the ICU for right heart catheterization. Initial
readings show a cardiac index of 1.8 L/min/m2, elevated pulmonary capillary wedge pressure (25
mmHg), and high pulmonary pressures (72/45 mmHg). Her initial BP was 105/55 mmHg, and she had a
heart rate of 105 beats per minute (bpm). Home medications include captopril, spironolactone,
metoprolol XL, and furosemide. Which of the following inotropic agents would be most appropriate?
a. Dobutamine
b. Milrinone
c. Dopamine
d. Isoproterenol 

161. One month ago, a 37-year-old woman with sinus infection responded well to a 14-day course of
amoxicillin/davulanate 875/125 mg twice daily. She is scheduled for a root canal in 1 week. In the past,
her dentist had prescribed one dose of clindamycin 600 mg, 1 hour prior to any dental work, for
endocarditis prophylaxis because of her history of mitral valve prolapse. Realizing she has not received
her prescription, the patient calls the dentist's office for an antibiotic.
What prophylaxis is indicated for this patient?
a. Azithromycin 500 mg PO1 hour before the procedure
b. Clindamycin 600 mg PO 1 hour before the procedure
c. No prophylaxis recommended in this patient
d. Amoxicillin 2 g PO 1 hour before the procedure

162. One year later this patient is in need of cholecystectomy. He remains asymptomatic. The surgeon
arranged for a TMT before you visited with the patient. The patient exercised to an equivalent of 9.0
METS with a normal HR and BP response. The ECG was interpreted as non diagnostic (<1 mm of up
slopping ST depression) at peak exercise and resolved by 3 min into recovery.
At this point, which of the following is true?
a. Repeat coronary angiography and possible coronary revascularization will improve the
patient's operative outcome
b. The operation should be postponed until an imaging stress test can be obtained
c. You should recommend IV beta blockers and IV NTG with PA catheter monitoring and to
proceed with the cholecystectomy
d. Repeat stress testing was not necessary at this point in time as the patient was active and
asymptomatic

163. Optimal time for elective non cardiac surgery after coronary stenting with bare metal stent:
a. Between 45 days to 180 days
b. Beyond 1 year
c. Between 180 days to 1 year
d. Within 45 days

164. A previously healthy 36-year-old African-American man is admitted from the emergency
department with newly diagnosed congestive heart failure. An echocardiogram is obtained on admission
and reveals significant four-chamber dilatation, an EF of 25%, and an LV end-diastolic dimension of 7.3
cm. A left heart catheterization reveals no evidence of obstructive coronary disease. The patient does
not use alcohol, cocaine, or other illicit drugs, and he has no recent history of viral illness. While
discussing his medical history, he notes that his father died of heart failure at age 38. You consider the
possibility that the patient has some form of familial cardiomyopathy.
Which of the following statements is TRUE concerning familial cardiomyopathies?
a. A thorough family history should be taken, including at least three generations.
b. Most familial cardiomyopathies exhibit autosomal recessive inheritance patterns.
c. It is estimated that <5% of idiopathic dilated cardiomyopathies are familial in origin.
d. To identify the accurate diagnosis, all family members should be tested first for multiple
mutations associated with the disease.

165. Pseudonormalization of filling pattern is seen in


a. Stage IV LV diastolic dysfunction
b. Stage II LV diastole dysfunction
c. Stage III LV diastolic dysfunction
d. Stage I LV diastolic dysfunction

166. R. W. is a 60-year-old woman with HF (left ventricular ejection fraction <30%) who has HTN with a
BP of 152/90 mmHg. Her potassium is 4.0 mg/dL and serum creatinine is stable at 1.5 mg/dL. She is
currently on digoxin and furosemide.
Which regimen is most appropriate to initiate in this patient?
a. Lisinopril 5 mg daily
b. Metoprolol tartrate 12.5 mg twice daily
c. Valsartan 20 mg twice daily
d. Hydralazine 25 mg four times daily

167. Randomized-controlled trial study that showed exercise training intervention after coronary
angioplasty can improve functional capacity and quality of life, lowered residual stenosis, decreased
events and hospital readmission significantly is...
a. O GOSPEL
b. ExTraMATCH
c. HF-action
d. EURO-ASPIRE
e. ETICA

168. Regarding calculation of LV mass, Relative Wall Thickness (RWT) ? 0.42 is considered:
a. Concentric remodeling if the LV mass index ? 115 (female) and ? 95 (male)
b. Either concentric remodeling or concentric hypertrophy if the LV mass index ? 95 (female) and
? 115 (male) c. Normal if the LV mass index ? 115 (female) and ? 95 (male)
d. Concentric hypertrophy if the LV mass index ? 95 (female) and ? 115 (male)
e. Eccentric hypertrophy if the LV mass index > 95 (female) and >115 (male)

169. Regarding echocardiographic score used to predict outcome of mitral balloon velocity, when the
Mobility: mid and base portion of the leaflet mobile normally. Subvalvular thickening: The thickening
chordate structure extending up to one-third of the chordal length Thickening: The valve thickening
extending through the entire leaflet and Calcification: some scattered areas of brightness confined to
leaflet margins, the mitral score will be:
a. 3-2-2-2
b. 2-2-2-2
c. 2-2-3-2
d. 2-3-3-2
e. 2-3-2-2

170. Regarding stress echocardiography, which of the following statement is appropriate to perform
stress echocardiography:
a. As a risk assessment following unstable angina/ non ST elevation MI without sign and
symptom of heart failure and not planning to undergo early catheterization
b. Asymptomatic patient with Agatson score greater or equal to 400
c. Exercise stress in patient with history of nonsustained ventricular tachycardia, Framingham
risk is moderate to high
d. Patient with chest pain with low pre test probability of CAD and the ECG is uninterpretable
e. All answer are correct

171. Regarding trans-thoracal echocardiography, which of the following statement is appropriate to


perform trans-thoracal echocardiography:
a. Initial evaluation in patient with murmur or click without other signs and symptoms of
structural heart disease
b. Routine surveillance (<1 y) of mild stenosis without change in clinical status/ cardiac exam
c. Routine perioperative evaluation of cardiac structure and function prior to noncardiac solid
organ transplantation
d. Routine surveillance (>3 y) of mild stenosis without change in clinical status/ cardiac exam
e. To evaluate arrhythmias in infrequent APC/ VPC without evidence of heart disease

172. Sick sinus syndrome in post op Congenital Heart Disease occurs most often in:
a. None of the above
b. TOF repair
c. Fontan and Glen
d. Arterial switch

173. the following data were obtained from a 75 year old man with a calcified aortic valve: left
ventricular outflow tract (LVOT) velocity (V1) 0.8 m/s, transaortic velocity (V2) 4 m/s, LVOT diameter 2
cm. the calculated aortic valve area (AVA) is:
a. 0.8 cm2
b. 0.4 cm2
c. 1 cm2
d. 0.6 cm2
e. 1.2 cm2

174. the following data were obtained from a 75 year old man with a calcified aortic valve: left
ventricular outflow tract (LVOT) velocity (V1) 0.8 m/s, transaortic velocity (V2) 4 m/s, LVOT diameter 2
cm.
True statement about this case include all the following EXCEPT:
a. Orthopnea, paroxysmal nocturnal dyspnea, and pulmonary edema are late manifestations of
this disorder
b. Gastrointestinal bleeding has been associated with this disorder • Syncope in this disorder
commonly occurs
c. syncope in this disorder commonly occurs without significant change in systemic vascular
tone
d. Syncope may have been due to an arrhythmia in this patient
e. Patient with this disorder who describe angina may not have significant coronary arterial
obstruction

175. The patient in the preceding question is diagnosed with acute pericarditis and treated successfully
with non steroidal anti-inflammatory agents. Over the next five years, she has recurrent episodes of
pericarditis, each treated with indomethacin. She now presents with the gradual onset of lower
extremity edema over six months. She has vague symptoms of exertional dyspnea and abdominal
bloating but doesn't feel acutely ill. When you see her in the office, she has obvious edema and
distended neck veins although her lung fields are clear. Her voltage on ECG is not significantly different
from her baseline, and an office echocardiogram suggests normal left ventricular function and no
significant effusion.
Which of the following findings would you expect to see during right heart catheterization?
a. Left ventricular end diastolic pressure of 25 mm Hg and a right ventricular end diastolic
pressure of 10 mm Hg
b. Right atrial pressure tracing with a steep Y descent
c. Increased cardiac output
d. Right atrial pressure of 3 mm Hg
e. Increased ventricular filling

176. There is some specific, supportive signs and quantitative parameters in grading of mitral
regurgitation. Which of following parameter / criteria is suitable for severe mitral regurgitation:
a. Vena contracta > 0.3 cm, with EROA 0.2 - 0.3 cm 2
b. Regurgitant jet hugging the entire LA wall (> 75% of LA)
c. Dens, parabolic CW Doppler MR signal
d. Regurgitant traction > 50%
e. Soft, triangular ON Doppler MR signal

177. which of the following disruptions of normal myocyte signaling occurs in heart failure ?
a. Increased levels of SERCA2a with calcium overload of the sarcoplasmic reticulum.
b. Internalization of the ryanodine receptor, making it unavailable on the surface membrane.
c. Use of T-type rather than L-type calcium channels to initiate contraction,
d. Uncoupling of the beta-1 adrenergic receptors from G proteins
e. Increase in the speed of cross-bridge cycling.

178. Which of the following drugs can significantly increase digoxin concentration?
a. Metoprolol
b. Simvastatin
c. Amiodarone
d. Fenofibrate

179. which of the following is not an AV node independent tachycardia?


a. Atrioventricular reentrant tachycardia
b. Atrial fibrillation
c. Atrial tachycardia
d. Atrial flutter

180. which of the following patients with an LVEF of 25% would be most appropriate to refer for a right
heart catheterization?
a. A 40-year-old woman with asthma and heart failure is admitted to the hospital. Her
examination is notable for blood pressure 150/80 mm Hg; pulse 110 bpm and regular; lungs
with diffuse expiratory wheezes; JVP <8 cm with no hepatojugular reflux; cardiac auscultation
with S12 and no S3; legs with no edema and are warm to touch.
b. An 85-year-old man with a 30-year history of diabetes previously received laser
photocoagulation therapy of diabetic retinopathy, and has ongoing leg pain from
neuropathy. He now presents with shortness of breath, has a blood pressure of 200/100 mm
Hg, anasarca, and a creatinine of 10.
c. A 20-year-old man is referred for cardiac transplantation due to his low LVEF despite 6
months of beta-blocker therapy. He reports mild fatigue while playing racquetball and his
examination shows blood pressure 120/80 mm Hg, pulse 72 bpm, JVP <8 with no
hepatojugular reflux, and there is no S3. Extremities are warm to touch, with no edema.
d. A 60-year-old man with an LVEF of 25% is admitted with shortness of breath and is found to
have blood pressure 130/80 mm Hg, pulse 90 bpm, and JVP of 14 cm with 2+ leg edema and
warm extremities, with creatinine 1.2.
e. A 30-year-old woman is admitted to the hospital with shortness of breath and a blood
pressure of 90/70 mm Hg. Following 2 days of intravenous diuretics, her examination is
notable for JVP of 16 cm and 2+ leg edema, with a systolic blood pressure of 78 mm Hg. Her
creatinine has increased from 1.6 to 2.5, with diuresis.

181. which of the following statements is true concerning the comparison of primary PCI and
thrombolysis in the treatment of AMI?
a. Survival is higher among elderly patients if treated with thrombolytic agents rather than
primary PCI
b. Survival with PCI in an 74-year-old diabetic female who presented 2 hrs after the onset of
MI with cardiogenic shock is likely to be better than with aggres- sive medical treatment
including thrombolysis
c. Survival benefit with primary PCI compared to thrombolysis is confined to anterior MI ©
Successful PCI and d. Survival benefit with primary PCI is not associated with operator volume
e. Success rates of rescue PCI after failed thrombolysis are similar to those of primary PCI
182. which of the following is the most common presenting symptom in patients with PAH?
a. Fatigue.
b. Chest pain.
c. Dyspnea.
d. Presyncope.

183. Which of the following tests would be appropriate to confirm a diagnosis of PAH in a patient with
elevated right ventricular systolic pressure demonstrated by echocardiography?
a. Liver function tests
b. Antinuclear antibody
c. Right Heart catheterization
d. Transesophageal enchocardiography

184. Which of the following would you recommend for a 19-year-old asymptomatic, nonobstructive
HCM patient with a maximal LV wall thickness of 18 mm, who is playing professional soccer?
a. Disopyramide.
b. ASA.
c. Discontinuation from organized, competitive sports.
e. EP study.

185. You are asked to review an ECG of a baby on the intensive care unit. The baby was well at birth, but
soon became unwell and cyanosed. There was no heart murmur. ECG findings reveal a superior axis,
absent right ventricular voltages, and a large P wave. What is the MOST likely diagnosis?
a. Complete atrioventricular septal defect
b. Tricuspid atresia
c. Critical pulmonary stenosis 0 Transposition of the great arteries
d. Total anomalous pulmonary venous connection (TAPVC)

186. You are asked to see a 50 year old female dietitian in consultation for HTN. She was found to have
an elevated BP on an medical check up 4 yrs ago. She followed her physician's recommendations and
uses only sodium substitutes, limits alcohol consumption, and exercises. She adopted a vegetarian
lifestyle. Despite these measures, her BP remained above normal and her health care provider
prescribed several medication regimens. However, her BP could not be maintained at <160/90mmHg.
Her medications include: Metoprolol: 25 mg twice daily, Lisinopril: 20 mg twice daily, Amlodipine: 10 mg
daily. Your examination detects the following: BP: 188/100 mmHg (seated), 190/100 mmHg (standing);
HR: 70 bpm, sitting, 80 bpm standing, Normal funduscopic examination, Normal peripheral pulses and
no abdominal bruits, Normal cardiopulmonary examination. This patient has continued medical therapy
and improves somewhat with addition of triamterene/HCTZ (37.5/25 mg) daily. Her BP is now 160
mmHg systolic. Laboratory results include: CBC: Normal Creatinine: 1.9 mg/dL Sodium: 145mEg/L
Potassium: 3.5mEg/L Uric acid: 3.0 mg/dL ECG: LVH by voltage criteria Chest X-ray: Normal
This patient has continued medical therapy and improves somewhat with addition of triamterene/HCTZ
(37.5/25 mg) daily. Her BP is now 160 mmHg systolic.
Laboratory results include:
CBC: Normal
Creatinine: 1.9 mg/dL
Sodium: 145mEg/L
Potassium: 3.5mEg/L
Uric acid: 3.0 mg/dL
ECG: LVH by voltage criteria
Chest X-ray: Normal
The most likely secondary form of HTN in this setting is:
a. Phenochromocytoma
b. Primary aldosteronism
c. Chronic renal failure
d. Renovascular stenosis
187. You are asked to see a 50 year old female dietitian in consultation for HTN. She was found to have
an elevated BP on a medical check up 4 yrs ago. She followed her physician's recommendations and uses
only sodium substitutes, limits alcohol consumption, and exercises. She adopted a vegetarian lifestyle.
Despite these measures, her BP remained above normal and her health care provider prescribed several
medication regimens. However, her BP could not be maintained at <160/90mmHg. Her medications
include: Metoprolol: 25 mg twice daily, Lisinopril: 20 mg twice daily, Amlodipine: 10 mg daily. Your
examination detects the following: BP: 188/100 mmHg (seated), 190/100 mmHg (standing); HR: 70 bpm,
sitting, 80 bpm standing, Normal funduscopic examination, Normal peripheral pulses and no abdominal
bruits, Normal cardiopulmonary examination.
Serum aldosterone concentration in this patient was 2 ng/dL (Normal: 1-21 ng/dL);
which of the following substances might be playing a role in this patient's HTN?
a. Angiotensin II
b. Premarin
c. Alcohol
d. Diuretic

188. You are asked to see a 50 year old female dietitian in consultation for HTN. She was found to have
an elevated BP on an medical check up 4 yrs ago. She followed her physician's recommendations and
uses only sodium substitutes, limits alcohol consumption, and exercises. She adopted a vegetarian
lifestyle. Despite these measures, her BP remained above normal and her health care provider
prescribed several medication regimens. However, her BP could not be maintained at <160/90mmHg.
Her medications include: Metoprolol: 25 mg twice daily, Lisinopril: 20 mg twice daily, Amlodipine: 10 mg
daily.
Your examination detects the following: BP: 188/100 mmHg (seated), 190/100 mmHg (standing); HR: 70
bpm, sitting, 80 bpm standing, Normal funduscopic examination, Normal peripheral pulses and no
abdominal bruits, Normal cardiopulmonary examination
Which of the following statements regarding the clinical presentation, which is correct?
a. The absence of an abdominal bruit excludes renovascular HTN as the underlying diagnosis
b. The next step should be US assessment of renal arterial flow
c. The BP response to postural change suggests a state of low volume-high resistance HTN
d. The HTN is not "resistant" because the patient is not taking appropriate medications at
their maximum doses

189. You are asked to see a 50 year old female dietitian in consultation for HTN. She was found to have
an elevated BP on an medical check up 4 yrs ago. She followed her physician's recommendations and
uses only sodium substitutes, limits alcohol consumption, and exercises. She adopted a vegetarian
lifestyle. Despite these measures, her BP remained above normal and her health care provider
prescribed several medication regimens. However, her BP could not be maintained at <160/90mmHg.
Her medications include: Metoprolol: 25 mg twice daily, Lisinopril: 20 mg twice daily, Amlodipine: 10 mg
daily. Your examination detects the following: BP: 188/100 mmHg (seated), 190/100 mmHg (standing);
HR: 70 bpm, sitting, 80 bpm standing, Normal funduscopic examination, Normal peripheral pulses and
no abdominal bruits, Normal cardiopulmonary examination. This patient has continued medical therapy
and improves somewhat with addition of triamterene/HCTZ (37.5/25 mg) daily. Her BP is now 160
mmHg systolic. Laboratory results include: CBC: Normal Creatinine: 1.9 mg/dL Sodium: 145mEg/L
Potassium: 3.5mEg/L Uric acid: 3.0 mg/dL ECG: LVH by voltage criteria Chest X-ray: Normal
This patient has continued medical therapy and improves somewhat with addition of triamterene/HCTZ
(37.5/25 mg) daily. Her BP is now 160 mmHg systolic.
Laboratory results include:
CBC: Normal
Creatinine: 1.9 mg/dL
Sodium: 145mEg/L
Potassium: 3.5mEg/L
Uric acid: 3.0 mg/dL
ECG: LVH by voltage criteria
Chest X-ray: Normal
According to this patient, the diagnosis of primary aldosteronism requires each of the following EXCEPT:
a. Suppressed plasma renin activity
b. Normal renal arteries
c. hypertension
d. Increased 24-hr urinary aldosterone rate
e. Hypokalemia (salt replete)

S-ar putea să vă placă și